Cardiology

¡Supera tus tareas y exámenes ahora con Quizwiz!

A 73-year-old man presents with worsening shortness of breath with activity over the last few months. He is not able to complete as many physical activities during the day as he could manage 3 months ago. Physical examination reveals hypotension, tachycardia, extremities that are cool to the touch, expiratory wheezing, and rhonchi auscultated during the pulmonary exam; there is also a diminished first heart sound with an S3 gallop heard during the cardiac exam. Question What would be the most critical lifestyle modification necessary for this patient? 1 Limiting salt 2 Limiting calories 3 Limiting fiber 4 Limiting physical activity 5 Limiting sleep

1. Limiting salt

A 5-year-old boy presents with a 6-day history of fever, fatigue, and rash. He has no significant past medical history. His vaccinations are current, except for varicella, which his parents have refused in the past. On exam, his temperature is 101.3°F, heart rate is 110, and blood pressure is 94/62. He has bilateral conjunctival injection, an erythematous pharynx without exudate, cracked red lips, and an erythematous right tympanic membrane. He has shotty enlarged anterior cervical lymph nodes bilaterally, the largest nodes measuring 1.6 cm on the right side and 1.5 cm on the left side. His lungs are clear, and his heart has a regular rhythm. His abdomen is soft. He is in no acute distress, and he has a generalized maculopapular rash. Question What other possible findings are associated with his probable diagnosis? 1 Sterile pyuria, meningismus 2 Peripheral neuropathy, thrombocytopenia 3 Epistaxes, hematuria 4 Colonic polyps, gastritis 5 Splenomegaly, pleuritis

1. Sterile pyuria, meningismus

A 66-year-old man presents with a history of recurrent episodes of chest tightness and shortness of breath that originally occurred with exertion but are now happening at rest. Dizziness also occurs with exertion. The pain is retrosternal, 6/10 in intensity, lasts for 10 minutes, and radiates to the neck, jaw, and shoulders. On examination, pulse is 65/min and low in volume; BP is 100/80 mm Hg. On auscultation, a loud mid-systolic murmur is heard at the second right intercostal space and radiates to the carotid arteries bilaterally. There is a single second heart sound. Question What is the most likely diagnosis? 1 Stable angina 2 Aortic stenosis 3 Pericarditis 4 Acute MI 5 Aortic dissection

2. Aortic stenosis

A 73-year-old man presents with worsening shortness of breath with activity over the last few months. He is not able to complete as many physical activities during the day as he could manage 3 months ago. Physical examination reveals hypotension, tachycardia, extremities that are cool to the touch, expiratory wheezing, and rhonchi auscultated during the pulmonary exam; there is also a diminished first heart sound with an S3 gallop heard during the cardiac exam. Question What medication should you begin in this patient to help alleviate his symptoms? 1 Statin 2 Antiarrhythmic 3 Diuretic 4 Vasodilator 5 Anticoagulant

3. Diuretic

A 47-year-old man presents for his annual physical exam. His past medical history is not significant, and he is not currently on any medications. He consumes 2 beers weekly and does not smoke. His blood pressure is normal during this visit. His primary care physician orders a fasting lipid panel with the following results: Component Value Triglyceride 135 HDL 36 LDL 220 Question In addition to diet and exercise, what medication should be started in this patient? 1 Cholestyramine 2 Fenofibrate 3 Nicotinic acid 4 Simvastatin 5 Ezetimibe

4. Simvastatin

A 65-year-old man presents to the office due to 6 months of bilateral buttock and thigh cramping pain. It occurs after walking 20 feet and is completely and quickly relieved with resting. His past medical history includes hypertension treated with atenolol, and he had a stroke 3 years ago. He also reports impotence for approximately the same duration of time. Question What is the patient's physical exam likely to include? 1 Absent femoral, popliteal, pedal pulses 2 Absent popliteal and pedal pulses; normal femoral pulses 3 Absent pedal pulses; normal femoral and popliteal pulses 4 Normal femoral, popliteal, pedal pulses 5 Normal femoral and popliteal pulses; diminished pedal pulses

1. Absent femoral, popliteal, pedal pulses

A 42-year-old woman with a history of dizziness and difficulty exercising presents seeking help. She reports fatigue, shortness of breath, and weakness for the past month. Upon physical exam, an EKG is conducted and an irregularly irregular heartbeat of 90 beats a minute. Laboratory findings showed no abnormal findings. Upon ECHO, there do not appear to be any abnormalities. Question What is the most likely explanation of the findings? 1 Atrial fibrillation 2 Congestive heart failure 3 Embolic stroke 4 Mitral valve regurgitation 5 Supraventricular tachycardia

1. Atrial fibrillation

A 47-year-old Caucasian woman presents for evaluation of a 4-week history of dyspnea. Symptoms started when on vacation in Colorado. She initially attributed the symptoms to the altitude, but she continued to have shortness of breath with mild activity after returning home: walking more than 100 feet, walking up one flight of stairs. Symptoms resolve with rest. She also reports mild exertional chest tightness and easy fatigability. She denies paroxysmal nocturnal dyspnea, orthopnea, edema, palpitations, and syncope. Past medical history includes usual childhood illnesses, no previous surgeries, and no known allergies. She takes a daily multivitamin and occasional Tylenol for headache. She does not smoke, rarely drinks alcohol, and denies use of recreational drugs. Physical exam shows temp 96.9°F, pulse 80 and regular, respirations 16, and BP 136/82. O2 sat is 96% on room air. The patient is an alert Caucasian woman in no acute distress, with no obvious jugular venous distention; non-labored respirations; lung fields clear to auscultation and percussion; and no rhonchi, rales, or wheezes. Heart shows RV heave present; normal S1 with fixed, split S2 with prominent P2 component; and grade II/VI systolic murmur at the left upper sternal border at the second intercostal space. The remainder of exam is within normal limits. CBC and BMP are unremarkable. Free T4 and TSH are within normal limits. EKG shows normal sinus rhythm with right ventricular hypertrophy, right atrial enlargement, and right axis deviation. There is an RSR in leads v1 and v2. Question What congenital heart defect does this patient most likely have? 1 Atrial septal defect 2 Coarctation of the aorta 3 Patent foramen ovale 4 Tetralogy of Fallot 5 Ventricular septal defect

1. Atrial septal defect

A 67-year-old man is recovering following an uncomplicated total hip arthroplasty. His past medical history is significant for hypertension and mild asthma. On postoperative day 2, he begins to note stiffness in his right leg. On exam, his right calf is noted to be swollen and slightly warmer than the left leg. The remainder of the exam is unremarkable. He is afebrile, and his vital signs are stable. Question What diagnostic evaluation is most appropriate in this situation? 1 Compression ultrasonography 2 Contrast venography 3 Contrast-enhanced MRI 4 Lower extremity X-ray 5 Serum B-type natriuretic peptide (BNP) measurement

1. Compression yltrasonography

A 43-year-old patient presents with a 3-hour history of acute onset of palpitations and dyspnea. There is no other significant medical history. Chest X-ray and echocardiography are normal. EKG reveals absence of P wave, with variable R-R interval with atrial rate of 300/min and ventricular rate of 120/min. Patient is afebrile, radial pulse is 90/min, BP is 110/70 mm Hg, and RR is 18/min. A diagnosis of newly detected atrial fibrillation is made. Question What is the next course of action? 1 Diltiazem 2 Propafenone 3 Observation 4 AV nodal ablation 5 Amiodarone

1. Diltiazem

A 65-year-old man with a 10-year history of hypertension controlled with lisinopril comes for preoperative evaluation before his arthroscopic knee surgery. He has never smoked and has never had a heart attack, congestive failure, arrhythmia, valvular problems, or abnormal lipids. His activity level is limited by knee pain; he does light to moderate housework and occasional swimming without dyspnea or chest pain. He is sexually active. His medications include ibuprofen for knee pain. His BMI is 26.5, blood pressure is 120/80 mm Hg, and the remainder of his exam is normal. His last electrocardiogram (EKG), taken 5 years ago shortly after a panic attack, showed normal sinus rhythm. His most recent (fasting) serum creatinine was 2.1 mg/dL with a blood urea nitrogen of 20 mg/dL. Past serum creatinines were in the 1.8-2.1 mg/dL range. He has no proteinuria. Question What further preoperative cardiac evaluation is indicated? 1 EKG 2 Cardiac catheterization 3 Holter monitoring 4 Echocardiography 5 Stress echocardiography

1. EKG

A 63-year-old woman presents with a blood pressure of 171/93 and returns for a follow-up appointment with a blood pressure of 181/94. The patient states she has a past medical history of alcohol abuse and high sodium intake. Additionally, the patient reports never using any antihypertensive medications. Upon physical exam, S2 heart sound is increased. The patient overall feels well besides her specific issue, and no other abnormal findings are found. Blood is not found in the urine, she presents with no flank pain, and she has a normal CT scan. Additionally, her T3 of 120 nanograms per deciliter and T4 levels of 5.0 micrograms per deciliter are in normal range. Question What is the most likely diagnosis? 1 Essential hypertension 2 Hypertensive emergency 3 Hyperthyroidism 4 Polycystic kidney disease 5 Resistant hypertension

1. Essential hypertension

A 42-year-old man has had systolic blood pressure in the 140s and diastolic blood pressure in the 80s on several occasions despite changing his diet and exercise regimen. His physician decides to start him on hydrochlorothiazide. Question What electrolyte abnormality symptoms should this patient be counseled on? 1 Hypokalemia 2 Hypoglycemia 3 Hypernatremia 4 Hypocalcemia 5 Hypermagnesemia

1. Hypokalemia

A 4-year-old boy presents with poor weight gain, small size for his age, and dyspnea upon feeding. His mother notes that the child suffers from frequent upper respiratory tract infections. On physical exam, the child is underweight for his age. You note a precordial bulge, a prominent right ventricular cardiac impulse, and palpable pulmonary artery pulsations. You also find a widely split and fixed second heart sound as well as a mid-diastolic rumble at the left sternal border. Question What pharmacologic agent would be most appropriate in the medical management of this patient at this time? 1 Lasix (furosemide) 2 Coumadin (warfarin) 3 Procardia (nifedipine) 4 Inderal (propranolol) 5 Indocin (indomethacin)

1. Lasix (furosemide)

A 79-year-old man presents with severe chest pain and dyspnea. He has a past medical history of diabetes mellitus, hypertension, and hyperlipidemia. He appears pale, apprehensive, and diaphoretic. He is in a confused state and agitated. His pulse is weak and tachycardic, with a systolic blood pressure of 60 mm Hg. He has a narrow pulse pressure, tachypnea, a weak apical impulse, and significant jugular venous distention. His lungs are free of crackles. Bedside electrocardiogram reveals ST-segment elevations in the anterior and septal leads. Question What concerning this patient's diagnosis is true? 1 Leading contributory cause is myocardial ischemia. 2 Profoundly reduced intravascular volume is responsible for hypotension. 3 Elevated serum lactate suggests improved prognostic rate. 4 Coronary artery angiography and revascularization is contraindicated. 5 High cardiac index and low pulmonary capillary wedge pressure are expected.

1. Leading contributory cause is myocardial ischemia.

A 50-year-old man with a history of DM and CKD presents to your office for a follow-up appointment for his hypertension. He reports some non-specific muscle weakness, so you decide to perform an ECG. The ECG demonstrates peaked T waves in several leads without any other abnormality. His labs reveal BUN 12, CO2 22, creatinine 1.0, Glucose 97, K 7.2, Cl 101, and Na 137. Question What medication is most likely to cause this ECG finding? 1 Lisinopril 2 Furosemide 3 Atenolol 4 Hydrochlorothiazide 5 Prazosin

1. Lisinopril

A 45-year-old man with no significant medical history presents with what he refers to as "anxiety attacks." He describes them as recurring bouts of palpitations, headaches, anxiety, and sweating that he has experienced for a few weeks. Last night, his wife witnessed him having an episode during dinner. She was concerned and immediately took his blood pressure: 195/105 mm Hg, so she brought him in today. On further questioning, he comments that he sometimes gets lightheaded when he stands up too rapidly; his mother had similar problems. On physical examination, his blood pressure is 165/90 mm Hg and his heart rate is 80 beats/min. A 24-hour collection of his urine test is positive for vanillylmandelic acid. Question What is the most likely diagnosis? 1 Pheochromocytoma 2 Essential hypertension 3 Cushing syndrome 4 Renal artery stenosis 5 Phenylketonuria

1. Pheochromocytoma

A 3-month-old male infant presents for a routine evaluation. His mother states that the child is gaining weight, is feeding appropriately, and has been without fever, chills, dyspnea or other abnormal objective signs. Upon physical examination, the examiner noticed a loud, harsh holosystolic murmur in the left third and fourth interspaces along the sternum that was associated with a systolic thrill. There were no other abnormalities. Question Given the most likely diagnosis, what patient education do you give this patient's mother? 1 The louder associated murmur is a positive sign. 2 This is a common finding in the adult population. 3 Symptoms of shortness of breath and difficulty feeding will occur. 4 This patient is expected to have a lower life expectancy. 5 Immediate surgical repair is warranted.

1. The louder associated murmur is a positive sign.

A 62-year-old woman is being treated for chronic congestive heart failure. She has been put on hydrochlorothiazide therapy. Her serum electrolyte levels are being monitored and show a persistent hypokalemia. Question What would be most appropriate to add to her therapeutic regimen? 1 Acetazolamide 2 Amiloride 3 Furosemide 4 Indapamide 5 Mannitol

2. Amiloride

A 32-year-old man with no significant past medical history presents with a 2-month history of increased dyspnea upon exertion; the dyspnea becomes apparent after walking 10 city blocks. He denies associated symptoms, such as fever, chills, changes in weight, chest pain, abdominal pain, nausea, and vomiting. He also denies any history of cigarette smoking, occupational risk factors, sick contacts, and recent travel. His physical exam reveals normal vital signs and no distension of his jugular vein, but there is a prominent right ventricular impulse along the lower-left sternal border that is associated with a palpable pulmonary artery. There is also a mid-systolic ejection murmur at the upper left sternal border that does not vary in intensity with respiration. There is a fixed split second heart sound. The remainder of his examination is normal. Question What is the most likely diagnosis? 1 Patent ductus arteriosus 2 Atrial septal defect 3 Ventricular septal defect 4 Mitral stenosis 5 Pulmonic stenosis

2. Atrial septal defect

A 6-week-old boy presents at your pediatric practice office, brought by his mother. She reports that for the past week he has not been feeding well and he breaks out into a cold sweat on his forehead while feeding. Upon further questioning, she reports that he becomes extremely breathless, irritable, and extremely pale after extended periods of crying. The mother reports a normal vaginal delivery and denies any problems with her son at the time of discharge from the hospital following his birth. She reports a family history of congestive heart failure. Vitals are as follows: pulse 130-regular, respiration 34-regular, blood pressure R arm 96/62 L arm 92/54 and R leg 70/42 L leg 74/40. Cardiac exam reveals 4+ carotid pulses bilaterally, 2+ brachial pulses bilaterally and absent femoral pulses bilaterally. A loud harsh systolic ejection murmur is noted at the base of the heart. The EKG reveals normal sinus rhythm with left ventricular hypertrophy. Question What is the most likely diagnosis? 1 Atrial septal defect 2 Coarctation of the aorta 3 Pulmonary stenosis 4 Tetralogy of Fallot 5 Ventricular septal defect

2. Coarctation of the aorta

A 1-month-old infant is evaluated for rapid breathing, feeding difficulty, lethargy, and poor weight gain. Physical exam is notable for tachypnea, tachycardia, a cardiac gallop, and a medium-pitched systolic murmur best heard posteriorly in the interscapular area with radiation to the left axilla, apex, and anterior precordium. A prominent anterior chest heave is also observed. The lower extremities demonstrate a 12 mm Hg pressure difference compared to the upper extremities. There are delayed femoral pulsations; upper extremity pulsations are normal. Question What is the best next step in management? 1 Blood pressure monitoring to compare upper extremities 2 Evaluation by a cardiovascular surgeon 3 Cardiac catheterizations for non-surgical approach 4 Initiation of anti-hypertensives 5 Omission of routine pre-surgery antibiotic prophylaxis

2. Evaluation by a cardiovascular surgeon

A 73-year-old man with no significant past medical history presents with a 1-month history of light-headedness, dizziness, and near syncope; it has been occurring in response to sitting up and standing from a supine position. He denies chest pain, palpitations, shortness of breath, cough, loss of consciousness, vision or speech changes, nausea or vomiting, numbness, tingling, paresthesias, and focal weakness. His physical exam is noteworthy for a drop in systolic blood pressure of 24 mm Hg from a supine to standing position. Question What clinical intervention should be recommended to this patient? 1 Begin clonidine or a diuretic 2 Fludrocortisone and compression stockings 3 Large, carbohydrate meal consumption 4 Sodium and water restrictions 5 Rise rapidly from seated and supine positions

2. Fludrocortisone and compression stockings.

You are currently on an inpatient pediatric hospitalist team; you see a preterm infant who has signs of failure to thrive. Other signs and symptoms found during the history and physical examination include tachypnea, bounding peripheral pulses, and a rough machine-like murmur. Question Considering the most likely diagnosis at this time, what clinical intervention should be initiated? 1 Ampicillin 2 Indomethacin 3 Prednisone 4 Furosemide 5 Dopamine

2. Indomethacin

A 15-year-old girl is referred to a cardiologist's office for workup of hypertension. Her mother reports a normal pregnancy and birth. There is no family history of heart disease. On physical exam, you note BP 140/70 left and right upper extremities, 90/70 left and right lower extremities, HR 85/min, RR 20/min. Brachial and femoral pulses are incongruent. You note pulsations in the suprasternal notch. Cardiac auscultation reveals a III/VI systolic ejection murmur. Question What would you expect to see on chest radiography? 1 Right ventricular hypertrophy and large pulmonary arteries 2 Left ventricular hypertrophy and a notch in the aorta 3 Boot-shaped heart with right ventricular prominence 4 Prominence of the aorta, pulmonary artery, and left atrium 5 Increased pulmonic vasculature

2. Left ventricular hypertrophy and a notch in the aorta

A 2-month-old female infant presents for a well-child visit. Her mother states that she is concerned about the patient's lack of interest in feeding and her rapid breathing spells. You acknowledge these concerns, and during the physical examination, you note severe tachypnea, bounding peripheral pulses, and a rough machinery murmur that is auscultated best near the second left intercostal space. Question What is the most likely diagnosis? 1 Atrioventricular septal defect 2 Patent ductus arteriosus 3 Coarctation of the aorta 4 Ventricular septal defect 5 Still murmur

2. Patent ductus arteriosus

A 22-year-old man presents with what he describes as a change in his heart rate. He indicates that his heart rate seems to speed up and then slow down for the past few hours. He adds no other symptoms. An EKG is ordered: irregular rhythm, a PR interval of 0.16 seconds, a P to QRS ratio of 1:1, heart rate of 75 bpm when exhaling and 86 bpm when inhaling, PP interval varies >0.12 seconds, and an RR interval that is noted to accelerate and decelerate during the respiratory cycle similar to the way the patient described. Question What is the most likely diagnosis? 1 Atrial flutter 2 Sinus arrhythmia 3 First-degree heart block 4 Third-degree heart block 5 Atrial premature beat

2. Sinus arrhythmia

A 15-year-old girl presents with a 1-hour history of rapid heartbeat, faintness, sweating, and nervousness. She is also experiencing shortness of breath and chest pain. The patient has no significant past medical history. There is no history of similar episodes. The patient is on no medications and she denies illicit drug use. On exam, BP is 70/60 mm Hg, and pulse is 200 bpm. RR is 22/min. She is afebrile, looks pale, and her palms are slightly sweaty. She is not comfortable sitting up, so she prefers lying down. She looks slightly apprehensive. Her heart and lung exam are negative except for the tachycardia; except for cool sweaty hands, a brief abdominal and extremity exam are non-revealing. The physician quickly places the paddles on the patient's chest to record the rhythm; this shows a narrow-complex regular tachycardia at 210 bpm. He requests oxygen, IV line, and continuous monitoring. An EKG is in the process of being completed. Question How should the patient's condition be managed? 1 Carotid sinus massage 2 Synchronized cardioversion 3 Adenosine 6 mg IV push 4 Diltiazem 10 mg IV push 5 Verapamil 5 mg IV push

2. Synchronized cardioversion

A 73-year-old man with no significant past medical history presents with a 1-month history of light-headedness, dizziness, and near-faintness; it has been occurring in response to sitting up and standing from a supine position. He denies chest pain, palpitations, shortness of breath, cough, loss of consciousness, vision or speech changes, nausea or vomiting, numbness, tingling, paresthesias, focal weakness, dysuria, or urinary frequency. His physical exam is noteworthy for a drop of systolic blood pressure of 24 mm Hg from a supine to standing position. Question What test is most helpful in identifying the cause of this patient's symptoms? 1 Hemoglobin A1c 2 Tilt-table test 3 Cardiac enzymes 4 CT scan of the head 5 Urinalysis

2. Tilt-table test

A 40-year-old man presents with irregular heartbeats over the course of several days. His past medical history is significant for the presence of mitral valve stenosis and atrial fibrillation (AF). He takes beta blockers regularly. His ECG shows atrial fibrillation with an irregular heart rhythm around 80 bpm. Question What do you prescribe to restore his sinus rhythm by cardioversion and prevent further complications? 1 Benzodiazepine 2 Warfarin 3 Digoxin 4 Amiodarone 5 Heparin

2. Warfarin

A 32-year-old woman is brought in via ambulance due to a fast heart rate. Her blood pressure is 114/76 mm Hg, and her heart rate is 156 bpm. She reports mild chest pain and shortness of breath. Her 12-lead EKG reveals retrograde P-waves that occur simultaneously with the QRS complexes but appear "hidden." Question After mechanical measures fail, what pharmacologic agent should be tried first? 1 Amiodarone 150 mg IV 2 Verapamil 5 mg IV 3 Adenosine 6 mg IV 4 Ibutilide 0.5 mg IV 5 Lidocaine 5 mg IV

3. Adenosine 6 mg IV

A 77-year-old Caucasian woman is admitted to the hospital for episodes of dizziness. Her symptoms have been present for approximately 4 months, and they have been progressively worsening. She has no dizziness when sitting or lying down. She does note some rapid heart rate associated with her dizzy spells. The dizziness becomes severe enough that she must sit down and is unable to proceed with walking, which is now significantly interfering with her functional activities. She denies associated chest pain, shortness of breath, or nausea. Past Medical History: Hypothyroidism, single episode of atrial fibrillation 2 years ago. Usual home medications: Aspirin 81 mg daily, Levothyroxine 88 mcg daily. Social History: Patient is married and has 2 adult children. She denies any history of smoking, alcohol use, or use of recreational drugs. Review of systems: Notable only for occasional cough, occasional headache, and mild anxiety. Vital signs: Temp 98.2°F, pulse 98 and regular, respirations 16, blood pressure 94/62 mm Hg (sitting, right arm). Question What is most likely to establish her diagnosis? 1 Assessment for pallor of the skin and nails 2 Assessment of skin turgor 3 Blood pressures lying, sitting, and standing 4 Cardiac auscultation 5 Romberg test

3. Blood pressures lying, sitting, and standing

A 55-year-old man presents with a 2-day history of confusion, increased respiratory rate, rapid pulse, notable malaise, thirst, and less-than-normal urination. In the morning, his wife gave him a multivitamin pill hoping that it would help; she tells you that he has long-standing hypertension and he injured his neck in a car accident 1 week ago. On examination, you find BP 92/41 mm Hg, HR 150, RR 35, rapid and weak pulse, cutis marmorata, cold edematous extremities, and bluish discoloration of the tongue and nails. Question What kind of shock does this patient probably have? 1 Neurogenic 2 Hypovolemic 3 Cardiogenic 4 Septic 5 Anaphylactic

3. Cardiogenic

A 77-year-old man presents with significant persistent hypotension that has been worsening for 1 hour. Further investigation of this patient reveals a significant increase in heart rate and substantial tachypnea. Urinary output is too minimal to measure, and upon quick physical examination, the patient's extremities are cool to touch. After beginning fluid resuscitation for shock protocol, the patient's symptoms appear to be worsening. Bedside ECG indicates ST elevation in leads II and III and aVF. Hemodynamic monitoring indicates large V waves on PCWP tracing, suggesting severe mitral regurgitation. Question Based on this scenario, how would this patient's shock be categorized? 1 Obstructive shock 2 Distributive shock 3 Cardiogenic shock 4 Septic shock 5 Hypovolemic shock

3. Cardiogenic shock

A 1-month-old full-term male infant has been diagnosed with tetralogy of Fallot. His disease is being classified as moderate, and he has been admitted to the neonatal intensive care unit for monitoring. He is now stable and is doing well. During a consultation between the infant's parents and the pediatric cardiologist, treatment options are being discussed. Question What is the pediatric cardiologist likely to recommend as definitive treatment? 1 Chronic oral β-blocking agents 2 No treatment; this disease is self-limiting 3 Closure of ventricular septal defect and pulmonary valvulotomy 4 Closure of atrial septal defect and aortic valvulotomy 5 Closure of atrial septal defect and pulmonary valvulotomy

3. Closure of ventricular septal defect and pulmonary valvulotomy

While rounding on a 2-hour-old male neonate at the hospital, his mother remarks that although her pregnancy and delivery were unremarkable and the baby was term, she has attempted to initiate breastfeeding even though the neonate "doesn't seem interested." On heart auscultation of the neonate, a late systolic murmur that radiates to the back is appreciated. Vital signs while the baby is awake are BP 90/50 mm Hg in the right arm and 58/42 mm Hg in the right lower extremity, HR 140, RR 40, axillary temp 37.0° C, and O2 sat 90% on room air. Question What is the most likely diagnosis? 1 Aortic stenosis 2 Ventricular septal defect 3 Coarctation of the aorta 4 Patent foramen ovale 5 Tetralogy of Fallot

3. Coarctation of the aorta

A 58-year-old man with recently diagnosed type 2 diabetes on metformin has developed Stage I hypertension over the past 3 months. When deciding what antihypertensive medication to begin for this patient, what is the primary reason for using an ACE inhibitor? 1 Aid in glycemic control and decrease insulin resistance 2 Decrease plasma volume and cardiac output 3 Delay the progression to end-stage renal disease 4 Decrease peripheral vascular resistance 5 Improve serum lipid profile

3. Delay the progression to end-stage renal disease

A 52-year-old woman presents for a routine checkup. She has two children, and she attained menopause 1 year prior to presentation. Pap smears, mammogram, and DEXA bone scan are normal. She is a non-smoker. Her previous biennial checkups were always normal. Her BP is 142/86 mm Hg, and pulse is 72 bpm. Her lab values are as follows: Fasting blood sugar 112 Postprandial blood sugar 138 Total cholesterol 190 LDL 102 TSH normal Question What is the next step in the management of this patient? 1 Reassurance 2 Thiazide 3 Diet and exercise 4 Statin 5 Metformin

3. Diet and exercise

A 57-year-old Caucasian man presents to the ED with syncopal spell 1 hour ago. 7 days ago, he developed acute diarrhea for 4 days, with progressively worsening nausea and vomiting since. Past medical history is significant for congestive heart failure secondary to non-ischemic cardiomyopathy, atrial fibrillation, hypertension, chronic renal insufficiency (with baseline creatinine 2.0), and BPH. He had laser surgery for a detached retina and transurethral retrograde prostatectomy. NKDA, but he indicates intolerance to diltiazem due to hypotension. Current medications: furosemide, digoxin, enalapril, carvedilol, tamsulosin. Decreased appetite for 3 days, fatigue, and malaise. He has been seeing yellow/green halos around lights for 3 days. He has chronic dyspnea on exertion at 3 blocks. Temp 98.8°F, pulse 40 and slightly irregular, respirations 16, blood pressure 108/60. Bradycardia is present with slightly irregular heart rhythm. Normal S1 and S2 without obvious rub, murmur, or gallop. Lungs fields are clear without rales, rhonchi, or wheezes. Lab: K+ 5.8 mEq/L, BUN 40 mg/dL, creatinine 4.2 mg/dL, digoxin level 4.8 ng/mL. EKG demonstrates complete AV dissociation present. CXR shows no acute disease. Abdominal X-ray is normal. Question What is the most likely diagnosis? 1 Complete heart block due to digoxin and beta blocker 2 Decompensated heart failure with right-sided symptoms 3 Digoxin toxicity secondary to renal failure 4 Sick sinus syndrome and viral gastroenteritis 5 Vasovagal syncope secondary to dehydration

3. Digoxin toxicity secondary to renal failure

A 55-year-old man presents for re-evaluation of his blood pressure; he has no significant past medical history. At his visit 3 weeks ago, his blood pressure was 145/90 mm Hg. He admits to somnolence, confusion, and non-specific bilateral visual disturbances over the past month; he denies any eye pain, blindness, ocular discharge, or floaters. His blood pressure today is 185/110 mm Hg. His fundoscopic exam reveals the following. Refer to the image. Question What is an additional expected manifestation in this case? 1 Scleral icterus 2 Rhinitis 3 Headache 4 Fruity breath 5 Erythema nodosum

3. Headache

A 76-year-old man presents with acute myocardial infarction. He quickly develops hypotension, altered mental status, cold clammy skin, and metabolic acidosis evident on laboratory tests. Question Based on the most likely diagnosis, what diagnostic study will be most helpful in demonstrating the severity of this patient's condition? 1 Complete blood cell count (CBC) 2 Arterial blood gas (ABG) 3 Lactate levels 4 Cortisol level 5 Blood culture

3. Lactate levels

A 22-year-old woman presents for a physical examination. When questioning her for medical history, you discover that she has a history of rheumatic fever. Upon listening to her heart, you detect a diastolic murmur. The murmur is low-pitched and begins with a loud snapping sound. Question What valvular abnormality is most likely present in this patient? 1 Mitral regurgitation 2 Aortic regurgitation 3 Mitral stenosis 4 Aortic stenosis 5 Tricuspid stenosis

3. Mitral stenosis

A 52-year-old patient with a known case of renovascular hypertension presents with poorly controlled hypertension. He has been treated with both enalapril and nifedipine. He had been diagnosed with unilateral left renal artery stenosis, but recent tests have demonstrated mild changes in the right renal artery also. Question What should be the next step in management? 1 Addition of diuretic 2 Reduction of NaCl intake 3 Percutaneous transluminal angioplasty 4 Addition of atorvastatin 5 Left nephrectomy

3. Percutaneous transluminal angioplasty

A routine EKG 5 weeks ago determined that a 59-year-old Caucasian man had developed new-onset atrial fibrillation. The atrial fibrillation was asymptomatic and was not associated with dizziness, palpitations, chest pain, or shortness of breath. He was started on warfarin 5 mg daily for anticoagulation and his Toprol dose was adjusted. Cardioversion was scheduled as a subsequent elective outpatient procedure. The patient's past medical history is notable for hypertension and coronary artery disease with prior RCA stent. He has no history of stroke or TIA. Allergies: bee stings. Usual home medications: aspirin 81 mg daily, hydrochlorothiazide 25 mg daily, metoprolol tartrate 50 mg b.i.d., warfarin 5 mg daily. He quit smoking 20 years ago. He now presents to the outpatient cath lab for a planned elective cardioversion procedure. Patient reports no new medical problems or symptoms since his diagnosis of new-onset atrial fibrillation. He claims compliance to his medical regimen. He denies chest pain or shortness of breath. His vital signs are stable. Physical exam reveals clear lung fields and an irregular cardiac rhythm. EKG shows atrial fibrillation with a ventricular rate of 87 beats per minute. Review of his lab work from 3 days prior shows the following: WBC 6.2 K/mcL Sodium 140 mEq/L Hgb 11.2 g/dL Potassium 4.8 mEq/L Hct 42% Chloride 102 mEq/L Platelets 342 K/mcL CO2 24 mEq/L Protime 16.2 seconds BUN 19 mmol/L INR 1.7 Creatinine 1.1 mg/dL Today's lab work shows protime 19.2 seconds and INR 2.1. Question What is the most appropriate course of action? 1 Perform pharmacologic (rather than electrical) cardioversion. 2 Perform transesophageal echocardiogram, reschedule cardioversion in 2 weeks. 3 Postpone cardioversion procedure. 4 Proceed with cardioversion. 5 Proceed with cardioversion after a dose of enoxaparin (Lovenox).

3. Postpone cardioversion procedure

You are called to the emergency department at 2 PM to see a 44-year-old male patient with a history of coronary heart disease. He smokes 3 packs of unfiltered cigarettes per day an has crushing chest pains and palpitations. He has a wide-complex rapid regular tachyarrhythmia at 160 beats per minute. When you reach his examination room, you note his monitor also reveals evidence of P-waves at 75 beats per minute. Question What type of rhythm do his symptoms show? 1 Paroxysmal supraventricular tachycardia 2 Sinus tachycardia 3 Ventricular tachycardia 4 Ventricular fibrillation 5 Atrial fibrillation

3. Ventricular tachycardia

A 66-year-old woman with a history of a multinodular goiter presents to the office after a recent hospitalization for a newly diagnosed cardiac arrhythmia. She relates that she has continued to see the cardiologist and is being treated with an oral medication. She denies any symptoms of hyper or hypothyroidism. She denies any change in the size of her gland or associated dysphagia or dyspnea. Thyroid function studies continue to remain in the normal range. Question When following this patient with multinodular goiter, what cardiac drug should be avoided or monitored closely? 1 Propranolol (Inderal) 2 Flecainide (Tambocor) 3 Sotalol (Betapace) 4 Amiodarone (Cordarone) 5 Mexiletine (Mexitil)

4. Amiodarone (Cordarone)

A 33-year-old man presents for an initial visit to a new primary care office. He has not seen a healthcare provider in the past 5 years. His past medical history reveals a coarctation of the aorta repair at age 13, after which he saw a cardiologist yearly until age 18. Since then, he has not had insurance and has only sought care for urgent problems in acute care clinics. Question What is a common complication of coarctation that primary care should regularly monitor this patient? 1 Anemia 2 Palpitations 3 Orthostatic hypotension 4 Hypertension 5 Hypertriglyceridemia

4. Hypertension

A 56-year-old man presents with primary hypertension. His additional medical history includes only nephrolithiasis. Past analysis of his kidney stones has revealed a calcium oxalate composition. Question What class of antihypertensives might best be avoided due to his history of nephrolithiasis? 1 Angiotensin-converting enzyme (ACE) inhibitors 2 Beta blockers 3 Calcium channel blockers 4 Loop diuretics 5 Thiazide diuretics

4. Loop diuretics

An 83-year-old woman presents to the emergency department due to dizziness, blurry vision, and weakness upon moving from laying down to a standing position. She states that she has been feeling like this for the past few weeks. She consumes two cups of coffee daily. She denies any feeling of a racing heart, breathing difficulties, or overwhelming feelings. Upon physical exam, her blood pressure measured 110/88 lying down. After 3 minutes of standing, a blood pressure reading of 90/78 was read, and a blood pressure of 92/80 sitting. Labs were within normal limits. Question What is the most likely diagnosis? 1 Benign positional vertigo 2 Beriberi 3 Caffeine overdose 4 Orthostatic hypotension 5 Panic attack

4. Orthostatic hypotension

A preterm female infant born to a 32-year-old woman with no known past medical illnesses presents for the infant's 1-week follow-up. The mother reports that the patient is behaving normally and is feeding well. The physical exam is remarkable for a murmur, which is located at the second left intercostal space. The murmur is continuous throughout cardiac systole; it is diastolic, non-radiating, and of a "machinery" quality. There is also a widened pulse pressure. The skin and mucosa are without cyanosis, and there is no evidence of fluid retention. Question Based on the physical exam finding, what is the most likely diagnosis? 1 Atrial septal defect 2 Ventricular septal defect 3 Tetralogy of Fallot 4 Patent ductus arteriosus 5 Pulmonary stenosis

4. Patent ductus arteriosus

A 43-year-old Caucasian woman, previously in good health, presented to the emergency department with headache, blurred vision, and dizziness. Symptoms started 3 days ago and progressively worsened. Past medical history: hypertension, hypothyroidism, prior cholecystectomy. No known drug allergies. Medications: HCTZ 25 mg daily, diltiazem CD 120 mg daily, and levothyroxine 88 mcg daily. She ran out of all medications 2 weeks ago. Vital signs were normal, except for blood pressure 210/114 in the right arm, 215/115 left arm, 220/100 right leg, and 215/112 left leg. Physical exam: Heart - no visible or palpable PMI; normal S1 and S2 without murmur, rub, or gallop. Pulmonary - few faint RLL crackles, which cleared upon coughing. Remainder of the physical exam, including neurologic exam, was unremarkable. CBC and BMP were unremarkable except for K+ 2.3 mEq/L. EKG - NSR with one PVC. Chest X-ray - clear lung fields; normal pulmonary vasculature. CT head - no evidence of intracranial pathology. Renal artery sonogram - Unremarkable. Patient was treated with IV nitroprusside and IV KCl 40 mEq x 2 doses and was admitted for further treatment. Over the next 2 days, patient's blood pressures gradually normalized with medical therapy, but potassium levels remained low despite treatment. Day Time K+ level Treatment Ordered 2 7:00 AM 3.2 40 mEq oral KCl 2 7:00 PM 2.8 20 mEq oral KCl x 3 doses, for total of 60 mEq 3 7:00 AM 2.7 20 mEq oral KCl x 3 doses, for total of 60 mEq 3 4:00 PM 3.1 -- Question What is the most likely underlying cause of her symptoms? 1 Cerebral aneurysm rupture 2 Coarctation of the aorta 3 Pheochromocytoma 4 Primary hyperaldosteronism 5 Renal artery stenosis

4. Primary hyperaldosteronism

A 6-week-old male infant presents with his mother. His mother reports that he has spells of turning "blue all over," especially when eating. On exam, he appears fatigued. He is noted to have clubbing of his fingers. Cardiac exam reveals a harsh systolic crescendo-decrescendo murmur in left upper sternal border. A chest X-ray reveals right ventricular hypertrophy. Question What is the most likely diagnosis? 1 Ventricular septal defect 2 Transposition of great vessels 3 Coarctation of the aorta 4 Tetralogy of Fallot 5 Atrial septal defect

4. Tetralogy of Fallot

A 70-year-old woman presents to the ER with a 1-week history of palpitations, dyspnea, and generalized weakness. She also gives history of decreased oral intake and weight loss. The patient has no significant previous medical history. On exam, the patient is afebrile. Pulse is 130/min, BP is 100/68 mm Hg, RR is 14/min, oxygen saturation of 97% on room air. Skin appears warm and smooth without cyanosis or edema. Cardiovascular exam reveals normal S1 and S2, no murmurs, rubs, or gallops. Lung sounds are clear bilaterally. Chest X-ray shows no acute cardiopulmonary disease. Electrocardiogram shows atrial fibrillation with rapid ventricular rate of 135 bpm. Normal QRS and QT intervals. Question What is the next step in management of this patient? 1 Echocardiogram 2 Sleep study 3 Chest CTA 4 Thyroid function tests 5 Adrenal functional tests

4. Thyroid function tests

A 3-month-old Caucasian male infant presents for a well-baby check. There have been no other changes since the last visit. Upon exam, the infant is pink and well-appearing. Cardiovascular exam reveals a grade III/VI high-pitched, harsh pansystolic murmur heard best at the left sternal border, fourth intercostal space (ICS). No additional murmurs are heard. The remainder of the exam is unremarkable. Question What is the most likely diagnosis? 1 Atrial septal defect 2 Coarctation of the aorta 3 Aortic regurgitation 4 Ventricular septal defect 5 Patent ductus arteriosus

4. Ventricular septal defect

A 66-year-old man with a past medical history of myocardial infarction 2 years ago, aortic regurgitation, congestive heart failure, atrial fibrillation, and chronic obstructive pulmonary disease is presently being monitored in the hospital. Myocardial infarction has been ruled out. An EKG performed upon admission revealed significant Q waves in the anterior leads but no evidence of an acute myocardial infarction. A diagnostic echocardiogram confirms moderate aortic and mitral valve regurgitation and a left-ventricular ejection fraction of 30%. He denies any complaints upon bedside evaluation. His physical exam reveals a blood pressure of 95/55 mm Hg and tachycardia. Continuous bedside ECG monitoring notes wide monomorphic QRS complexes with a heart rate of 160 beats per minute that spontaneously resolve within 20 seconds, reverting to the pattern identified upon admission. Question What antiarrhythmic agent would be the pharmacologic treatment of choice in the management of this patient? 1 Lidocaine 2 Procainamide 3 Verapamil 4 Calcium 5 Amiodarone

5. Amiodarone

A 48-year-old man presents for an annual physical exam. He has a past medical history of obesity and a 5-year history of hypertension that is currently not well-controlled. He was also recently diagnosed with type 2 diabetes mellitus (DM). He is a 20 pack-year smoker and drinks 2-3 beers per night. On exam, his BMI is 41, and BP is 145/92 mm Hg. The remainder of his exam is unremarkable. Question What is the most important matter to address with this patient concerning the prevention of morbidity and mortality? 1 Obesity 2 Alcohol use 3 Lack of exercise 4 Hypertension 5 Cigarette smoking

5. Cigarette smoking

A 3-month-old male infant is brought into the pediatric clinic for assessment. The mother states that her child is not gaining adequate weight despite a regular breastfeeding schedule. She notes that the child appears to get "very tired and inactive" during and after feedings and that she can feel copious amounts of sweat on the child's skin following feedings. She denies any known illness in her child and recalls a normal birth. The general survey reveals a weight and length in the 40th percentile, tachycardia, and tachypnea. The cardiac exam is remarkable for a bounding and hyperdynamic precordium, a holosystolic harsh murmur audible over the lower sternal border (LSB), and a loud second component of the second heart sound. Question What is the best diagnostic tool to confirm the diagnosis? 1 Chest radiograph 2 Electrocardiogram 3 Cardiac catheterization 4 Magnetic resonance imaging 5 Echocardiogram

5. Echocardiogram

A 5-day-old female newborn was born 5 weeks prematurely and presents to her first pediatrician's appointment. She did not have any feeding or breathing issues, so mother and child had only a 2-day stay at the hospital. During the cardiovascular examination, the pediatrician notes that the newborn has a distinct murmur with a rough machine-like quality that is maximal at the second intercostal space at the left sternal border. The murmur starts after S1 and passes through S2 into diastole. Question What study is the first choice to confirm it the most likely diagnosis? 1 Electrocardiography 2 Chest X-ray 3 Cardiac catheterization 4 Chest CT scan 5 Echocardiography

5. Echocardiography

A 74-year-old man with a history of diabetes mellitus, hypertension, and hyperlipidemia presents with severe chest pain and dyspnea. On exam, he is confused, agitated, pale, apprehensive, and diaphoretic. His pulse is weak and tachycardic; systolic blood pressure is 80 mm Hg. He has a narrow pulse pressure, tachypnea, a weak apical impulse, significant jugular venous distention, and pulmonary crackles. Bedside electrocardiogram reveals ST-segment elevations in the anterior and septal leads, while a portable chest X-ray notes diffuse pulmonary congestion. Question What is the most appropriate step in the management of this patient? 1 Immediate crystalloid infusion 2 Initiate intravenous beta blocker therapy 3 Begin urgent phenylephrine 4 Quickly administer nitrates and morphine 5 Emergent percutaneous coronary intervention

5. Emergent percutaneous coronary intervention

A 48-year-old man presents with a logbook of home-recorded blood pressure readings between 125-185/75-100 mm Hg. His past medical history is significant for hypertension and obesity (BMI 31 kg/m2). He is a non-smoker and does not drink alcohol. He has been adhering to a sodium-restricted diet and tries to walk 2 miles at least twice a week. For the past 2 years, he has been taking hydrochlorothiazide 12.5 mg daily with no apparent side effects. Physical exam is unremarkable, and vital signs reveal a heart rate of 85 beats per minute and blood pressure of 150/85 mm Hg. Question Based on this information, what is the most appropriate next step in the management of the patient's hypertension? 1 Discontinue hydrochlorothiazide and begin a calcium channel blocker. 2 Change hydrochlorothiazide 12.5 mg daily to chlorothiazide 125 mg daily. 3 Continue current management and schedule a follow-up visit in 3 months. 4 Discontinue hydrochlorothiazide and begin an ACE inhibitor. 5 Increase hydrochlorothiazide dose to 25 mg daily.

5. Increase hydrochlorothiazide dose to 25 mg daily

A 2-week-old female neonate presents for her scheduled newborn visit. The mother notes that the newborn has been feeding poorly and seems to have difficulty catching her breath when crying. On examination, a continuous machine-like murmur is heard at the left first intercostal space. Question What is the most likely diagnosis? 1 Atrial septal defect 2 Ventricular septal defect 3 Coarctation of the aorta 4 Pulmonary stenosis 5 Patent ductus arteriosus

5. Patent ductus arteriosus

A newborn child is routinely evaluated in the pediatrician's office 1 month after delivery. The mother reports that the patient is behaving normally and is feeding well. The physical exam is remarkable for a murmur, which is located at the second left intercostal space. The murmur is continuous throughout cardiac systole and diastole, non-radiating, and of a "machinery" quality. There is additionally a widened pulse pressure. The skin and mucosa are without cyanosis, and there is no evidence of fluid retention. Question What is the most likely diagnosis? 1 Ventricular septal defect 2 Atrial septal defect 3 Transposition of the great arteries 4 Tetralogy of Fallot 5 Patent ductus arteriosus

5. Patent ductus arteriosus

A 72-year-old man is admitted with an acutely severe myocardial infarction. His status quickly deteriorates. His symptoms include hypotension, altered mental status, cold clammy skin, and metabolic acidosis seen on laboratory tests. Question What is the most appropriate initial pharmaceutical choice for a patient in this type of shock? 1 Aspirin 2 Clopidogrel 3 Lidocaine 4 Beta blockers 5 Vasopressors

5. Vasopressors

A 16-year-old boy with no significant past medical history presents to learn the results of a lipid panel that was performed. The lipid panel was ordered due to periorbital and extensor tendon xanthomas on the patient's body. The patient consumes a low-fat and low-calorie diet, and he exercises daily. He denies any bothersome symptoms, and his physical exam is otherwise unremarkable. Question What is the most likely mechanism for this patient's metabolic disorder? 1 Increased production of high-density lipoprotein 2 Undiagnosed diabetes mellitus 3 Hypersecretion of triglyceride-carrying chylomicrons from the small intestine 4 Reduction in the amount of LDL receptors on hepatocytes 5 Increased VLDL synthesis by the liver

4. Reduction in the amount of LDL receptors on hepatocytes

A 68-year-old woman presents with shortness of breath, fatigue, dry cough, and swelling in her ankles. Symptoms started around the time of her son's wedding 6 weeks ago. She dismissed them as stress-related, but symptoms have worsened in intensity and frequency. Now she becomes very short of breath with any exertion, such as climbing stairs in her home. She feels like she may pass out and has to sit when she gets lightheaded. Her breathing is more difficult when lying down. She denies productive cough, fever, or chest pain. She has had no medical care for several years. Her past medical history is unremarkable, with no known medical conditions and no surgeries. She is menopausal and does not take any medications, nor does she have any allergies. She denies the use of tobacco, alcohol, and drugs. Vitals are shown in the table. Physical exam was performed by the provider on an earlier shift, and the patient's care was turned over to you. Weight 171 lb Height 65" BMI 28.5 Pulse 101 Blood pressure 158/98 Temperature 97.9°F/36.6°C Several test results are available and shown. Complete blood count (CBC) Normal Thyroid stimulating hormone (TSH) Normal Urinalysis Normal Complete Metabolic Panel (CMP) Normal Beta natriuretic peptide (BNP) Elevated Chest X-ray Perivascular edema, slightly enlarged cardiac shadow 12-lead electrocardiogram Mild left ventricular hypertrophy Cardiac echocardiogram Pending Cardiac enzymes Normal Question Once this patient is stabilized and educated about her diagnosis and treatment, she asks what she could have done to prevent her condition. What is the greatest modifiable contributing factor to this condition at the population level? 1 High dietary sodium intake 2 Inability to maintain ideal body weight 3 Lack of annual exams 4 Sedentary lifestyle 5 Untreated hypertension

5. Untreated hypertension

A 28-year-old African American man presents with dyspnea associated with mild substernal chest pain and dizziness. Symptoms are provoked by sporting activities, and they are relieved with rest. His physical exam reveals a harsh murmur best heard at the left lower sternal border; it decreases in intensity upon squatting. There is also an S4 gallop. A bedside electrocardiogram was remarkable for left ventricular hypertrophy and septal Q waves in the anterolateral leads. An echocardiogram noted asymmetric LVH and a septum that was twice the thickness of the posterior wall. Question What would be the next step in the management of this patient? 1 Begin this patient on a beta-adrenergic blocker. 2 Recommend increased aerobic activity to improve conditioning. 3 Refer this patient for immediate septal myectomy. 4 Implant a dual-chamber pacemaker. 5 Initiate oral digitalis, diuretic, and nitrate therapy.

1. Begin this patient on a beta-adrenergic blocker.

A 68-year-old man with a history of hypertension, hyperlipidemia, and myocardial infarction presents to the emergency room with a 3-day history of shortness of breath at rest. He has found it difficult to walk short distances due to shortness of breath. He reports orthopnea, nocturnal dyspnea, and generalized abdominal discomfort. He denies cough, fever, chills, diaphoresis, anxiety, chest pain, pleurisy, cough, nausea, vomiting, diarrhea, rashes, lightheadedness, and syncope. Upon physical examination, the patient is acutely dyspneic. He is afebrile but tachypneic and diaphoretic. There is a diminished first heart sound, S3 gallop, and laterally displaced PMI. The abdominal exam reveals distension with hepatomegaly in the right upper quadrant. There is 2+ pitting edema of the lower extremities to the level of the mid-calf. A bedside chest X-ray revealed pulmonary vascular congestion and Kerley B lines. Question What additional finding is expected in this patient? 1 Bibasilar rales 2 Hyperresonance to percussion 3 Warm and dry skin 4 Depressed jugular venous pressure 5 Bradycardia

1. Bibasilar rales

A 50-year-old man presents with chest pain. The pain is substernal, described as sharp, worsens with inspiration, and improves with leaning forward. A pericardial friction rub is appreciated on auscultation. Question What EKG finding is expected with this patient's condition? 1 Diffuse ST elevation 2 S1Q3T3 pattern 3 Delta waves 4 U waves 5 Sine wave pattern

1. Diffuse ST elevation

A 49-year-old woman presents due to gradual onset of reduced exercise tolerance while working out at the gym. She is afebrile and otherwise feels well. Cardiac exam reveals a III/VI diastolic rumbling murmur located at the apex; it is heard best in the left lateral position. Question What is the most likely diagnosis? 1 Tricuspid stenosis 2 Mitral stenosis 3 Mitral valve prolapse 4 Pulmonic stenosis 5 Aortic regurgitation

2. Mitral stenosis

A 36-year-old woman presents with chronic dyspnea that is worse while lying prone. The patient reports progressive worsening of the symptoms. On physical examination, a heart murmur is detected upon cardiac auscultation, heard best with the bell over the apex. The murmur is a non-radiating low-pitched diastolic rumble. A loud S1 and opening snap can also be heard in addition to an apical thrill and decreased pulse pressure. An EKG is done and shows atrial fibrillation. Question What is the patient's most likely underlying condition? 1 Aortic regurgitation 2 Pulmonic stenosis 3 Mitral stenosis 4 Hypertrophic subaortic stenosis 5 Mitral valve prolapse

3. Mitral stenosis

A 72-year-old man presents with "being short of breath"; he feels as though his belly is bloated and his legs are swollen. Past medical history includes high blood pressure (for which he is currently taking lisinopril) and high cholesterol controlled with diet modifications. The patient also has a past diagnosis of mediastinal lung cancer around 5 years ago for which he received radiation treatment as part of his prescribed therapeutic regimen. Physical examination reveals an elevated jugular venous pressure and Kussmaul sign. Moderate pitting edema and ascites are also observed. Chest radiograph reveals only mild cardiomegaly. Echocardiogram reveals a normal left ventricle chamber size, normal LVEF, and a thickened atrial septa. Question What is the most likely diagnosis? 1 Angina pectoris 2 Dilated cardiomyopathy 3 Essential hypertension 4 Hypertrophic cardiomyopathy 5 Restricted cardiomyopathy

5. Restricted cardiomyopathy

A 64-year-old Caucasian woman who takes no medications and has had no recent medical attention presents to establish care. She says high cholesterol "runs in her family," and she admits to eating fast food for lunch most days of the week. Question In light of her family history and current habits, what might be incidentally found on her physical exam? 1 Acanthosis nigricans on her neck 2 Buffalo hump on her back 3 Lipoma on the trunk 4 Systolic heart murmur 5 Xanthelasma over both eyes

5. Xanthelasma over both eyes

A 60-year-old man is 3 years post-coronary artery bypass grafting. His last lipid profile showed: Lipid Result Triglycerides 165 mg/dL HDL 34 mg/dL LDL 135 mg/dL He is normotensive and almost free of chest pain on isosorbide mononitrate and metoprolol. Question Given his lipid profile, how can this patient most appropriately be managed? 1 Fenofibrate, 134 mg daily 2 Simvastatin, 20 mg daily 3 Atorvastatin, 80 mg daily 4 Pravastatin, 20 mg daily 5 Lovastatin, 10 mg daily

3. Atorvastatin

A 70-year-old woman presents with a 3-day history of shortness of breath at rest. She has been finding it difficult to walk short distances due to shortness of breath. Additionally, she is experiencing orthopnea and nocturnal dyspnea. Her past medical history is significant for hypertension, hyperlipidemia, and myocardial infarction. The patient denies cough, fever, chills, diaphoresis, anxiety, chest pain, pleurisy, cough, nausea, abdominal pain, vomiting, diarrhea, rashes, lightheadedness, and syncope. Upon physical examination, the patient is short of breath; she requires numerous pauses during conversation. She is afebrile but tachycardic and diaphoretic; her extremities are cool. The exam reveals a diminished first heart sound, S3 gallop, laterally displaced PMI, bibasilar rales, dullness to percussion, and expiratory wheezing. There is no JVD noted, but 2+ pitting edema of the lower extremities to the level of the mid-calf is evident. Question What is the most likely diagnosis? 1 Congestive heart failure 2 Pulmonary embolism 3 Myocardial infarction 4 Chronic obstructive pulmonary disease 5 Bacterial pneumonia

1. Congestive Heart failure

A 14-year-old boy is seen for a sports physical for the freshman basketball team. Past history is significant for a high degree of myopia bilaterally (first diagnosed at age 4) and a dislocated shoulder at age 10 that was easily reduced. Family history is significant for several unidentified ancestors having died in their 40s of an unidentified cardiovascular disorder. Physical examination revealed normal vital signs. Height is 6'1" and weight 145 lb. The upper to lower segment ratio is 0.65 (decreased). Arm span was 76". The palate is highly arched and mild pectus excavatum is present. A 2/6 early diastolic murmur is present and best heard at the second intercostal space at the right sternal border. Arachnodactyly of the fingers and toes and generalized loose jointedness and pes planus are also present. Question What should be advised? 1 Echocardiography needs to be performed with follow-up. 2 Family history needs to be further elucidated. 3 He cannot play basketball due to the prior shoulder dislocation. 4 Heart murmurs are a relative contraindication to participation in sports. 5 There is no contraindication to his playing basketball.

1. Echocardiography needs to be performed with follow-up

A 55-year-old man with no significant past medical history presents for a routine evaluation and fasting bloodwork. He does not note any symptoms at this time. His physical examination reveals an obese body mass index with a waist circumference of 120 cm and a blood pressure of 140/90 mm Hg. Physical exam is otherwise unremarkable. His fasting bloodwork is drawn. Question What laboratory finding would qualify a diagnosis of metabolic syndrome in this patient? 1 HDL value of 35 mg/dL 2 Total cholesterol of 230 mg/dL 3 Triglyceride value of 125 mg/dL 4 LDL measurement of 110 mg/dL 5 Fasting plasma glucose of 95 mg/dL

1. HDL value of 35 mg/dL

A 26-year-old woman presents after a syncopal episode. She has lost of consciousness 3 times over the past 12 months. Each event occurred during or just after physical exercise. On PE: BP 110/70 mm Hg, HR 75/min, normal S1/S2, and a III/VI systolic ejection murmur is heard best at the left sternal border that decreases with squatting. The EKG shows a normal sinus rhythm with diffuse increased QRS voltage. Question What is the most likely diagnosis? 1 Hypertrophic cardiomyopathy 2 Mild mitral valve insufficiency 3 Moderate pulmonary stenosis 4 Severe aortic stenosis 5 Moderate mitral valve stenosis

1. Hypertrophic cardiomyopathy

A 72-year-old woman presents with a severe unilateral headache, jaw pain, and scalp tenderness. The patient states the headache is of a piercing quality; her jaw hurts only when she chews, and feels better a few minutes after she stops chewing. A pulsation of the temporal artery on the same side as her headache cannot be appreciated, and prednisone is prescribed until the patient can see a specialist. Question What is the reason prednisone is prescribed in this case? 1 Risk for blindness 2 Stenosis of the contralateral temporal artery 3 As treatment for pseudotumor cerebri 4 Edema reduction of berry aneurysm 5 Angioedema in the oropharynx

1. Risk for blindness

A 68-year-old woman presents with shortness of breath, fatigue, dry cough, and ankle swelling. Symptoms started 6 weeks ago; she dismissed them as stress-related, but they have worsened in intensity and frequency. She becomes short of breath with any exertion, such as climbing stairs. She feels like she may pass out and has to sit when she gets lightheaded. Her breathing is worse when lying down. She denies productive cough, fever, or chest pain. She notes she has had no medical care in the past few years and is post-menopausal. She denies any other past medical or surgical history. Social history is unremarkable. Vitals are notable for pulse 101, BP 158/98, and BMI 28.5. Exam reveals bibasilar crackles and 2+ pitting edema of the lower extremities. Complete blood count (CBC) Normal Thyroid-stimulating hormone (TSH) Normal Urinalysis Normal Complete metabolic panel (CMP) Normal Beta naturetic peptide (BNP) Elevated Chest X-ray Perivascular edema, slightly enlarged cardiac shadow 12-lead electrocardiogram Mild left ventricular hypertrophy Cardiac echocardiogram Pending Cardiac enzymes Normal Question Based on the most likely diagnosis, what medication is most appropriate for long-term therapy? 1 Lisinopril 2 Salmeterol 3 Diltiazem 4 Omega-3 fatty acids 5 Digoxin

1. Lisinopril

A 57-year-old woman presents with pain and swelling in her left leg. Her chart shows a history of osteoarthritis of the knees, mild hypertension, and type 2 diabetes mellitus that is controlled on medication. She was well until 1 week ago when she noted a bulge behind her left knee. Yesterday, she woke up with pain, redness, and swelling in her calf and stayed in bed most of the day with her leg propped up on a pillow. She has had no fever or chills, no new pain or swelling in her right leg, no shortness of breath, and does not otherwise feel ill. She denies recent travel or immobility. Vital signs are within normal limits. Significant on her physical exam is a red warm swollen left calf with trace pitting. There is no swelling in the thigh or toes, feet are warm and pedal pulses are intact. The left calf is tender to palpation posteriorly and is 1 cm larger than the right calf on measurement. The rest of her exam is unremarkable. A sensitive assay D-dimer blood test is negative. Question Based on this information, what is the likelihood of deep vein thrombosis and the next step in the care plan? 1 Low due to negative sensitive D dimer test. Consider ultrasound to further evaluate knee. 2 Low, but patient is at high risk for DVT. Order doppler ultrasound studies; if negative, repeat in a week. 3 Low; symptoms are more consistent with erysipelas. Order dicloxacillin 500 mg four times a day. 4 High since D-dimer tests have poor negative predictive value. Order compression ultrasound. 5 High, as indicated by swelling and tenderness on exam. Order immediate anticoagulation.

1. Low due to negative sensitive D dimer test. Consider ultrasound to further evaluate knee.

A 76-year-old man presents with substernal chest pain; it is associated with progressive exertional dyspnea, easy fatigability, and dizziness. These symptoms are exacerbated by walking short distances, and they are relieved with rest. He denies fever, chills, cough, wheezing, pleurisy, calf pain, abdominal problems, peripheral edema, cigarette, drug use, alcohol use, sick contacts, or travel. His physical exam reveals a normal blood pressure and a rough, harsh, low-pitched crescendo-decrescendo systolic murmur beginning after the first heart sound; it is best heard at the second intercostal space in the right upper sternal border. Its intensity is increased toward midsystole; the murmur radiates to both carotid arteries and is accentuated upon squatting, and it is reduced during Valsalva strain. His lungs are without adventitious sounds. Question What medication is most appropriate for management of this patient? 1 Metoprolol 2 Enalapril 3 Nitroprusside 4 Furosemide 5 Lanoxin

1. Metoprolol

A 35-year-old Costa Rican woman emigrated to the United States 5 years ago. She presents to the office due to chronic progressive dyspnea. She reports no chest pain, fever, or cough and is a non-smoker. Her daily activities are becoming increasingly limited due to her shortness of breath. On auscultation, she has an accentuated S1, a loud opening snap, and a diastolic murmur heard best at the apex in the left lateral decubitus position. She has 1+ ankle edema bilaterally. Question What is the most likely diagnosis? 1 Mitral stenosis 2 Mitral valve prolapse 3 Tricuspid stenosis 4 Aortic stenosis 5 Aortic regurgitation

1. Mitral stenosis

A 22-year-old woman presents due to palpitations. She denies chest pain and shortness of breath; she has not had any recent infections. Aside from a tonsillectomy as a child, she has no significant past medical history. Examination reveals a thin woman in no acute distress. Lungs are clear to auscultation bilaterally. Cardiac exam reveals a mid-systolic click. Question What is the most likely diagnosis? 1 Mitral valve prolapse 2 Mitral stenosis 3 Mitral regurgitation 4 Aortic stenosis 5 Aortic regurgitation

1. Mitral valve prolapse

A 24-year-old woman with a past medical history of mild scoliosis presents with palpitations, occasional chest pain, and dizziness upon standing from a supine position. Her symptoms have been ongoing for a while, but the patient's parents finally convinced her to come be evaluated. On cardiac auscultation, a mobile mid-to-late systolic click and a late systolic murmur heard best at the apex is noted; no other abnormalities are found. Blood pressure was 112/68 mm Hg. Question What is the most likely diagnosis? 1 Mitral valve prolapse 2 Aortic stenosis 3 Dehydration 4 Atrial myxoma 5 Atrial fibrillation

1. Mitral valve prolapse

A 70-year-old African American man presents to his primary care provider with a complaint of gradual but progressive dyspnea and fatigue on exertion. He notes difficulty in climbing stairs with associated lightheadedness, increased abdominal girth, and swollen bilateral lower extremities. He admits to chronic alcohol use but denies any heart disease, chest pain or pressure, diaphoresis, palpitations, a history of diabetes, cigarette smoking, or claudication. His physical exam reveals sinus tachycardia, bibasilar rales, a laterally-displaced PMI, an elevated JVP, an S3 gallop, a mitral regurgitation murmur, peripheral edema, and abdominal ascites. Question What is the next step in the management of this patient? 1 Obtain an echocardiogram. 2 Refer the patient for cardiac catheterization. 3 Perform a cardiac biopsy. 4 Order coxsackie and adenovirus serological titers. 5 Order cardiac troponin levels.

1. Obtain an echocardiogram

A 29-year-old woman presents with a previous history of mitral valve prolapse with murmur of regurgitation confirmed on echocardiogram with prosthetic valve replacement 1 year ago. Question Based on the recommendations by the American Heart Association, antibiotic prophylaxis would be recommended if your patient was scheduled for what procedure? 1 Periodontal disease treatment 2 Post-operative suture removal 3 Orthodontic appliance adjustment 4 Intracanal endodontic treatment 5 Taking of oral impression

1. Periodontal disease treatment

A 49-year-old Caucasian man well known to your practice presents due to his history of hypertriglyceridemia. He seeks evaluation of his recent cholesterol lab values. He has a significant family history of cardiovascular disease; his mother had a heart attack at age 57, and his father had open heart surgery at age 60. The patient has had low HDL levels in several past cholesterol screening tests. At the last office visit, lifestyle modifications were implemented by the patient. He has been extremely conscientious about his diet; for the past year, he has also been participating in physical activity 6 days a week. In addition, he has significantly limited his alcohol intake. Current fasting lab values for the patient are as follows: total cholesterol of 235 mg/dL, triglycerides of 350 mg/dL, HDL of 35, and an LDL of 175 mg/dL. You decide to initiate pharmacologic therapy to treat the patient's cholesterol and triglyceride levels. Question What medication should you prescribe? 1 Rosuvastatin 2 Ezetimibe 3 Colestipol 4 Simvastatin 5 Cholestyramine

1. Rosuvastatin

A 75-year-old African American man presents with a 5-month history of gradually progressive dyspnea that is especially pronounced when climbing stairs. He also has been noticing that his ankles and lower legs have "gotten larger" over roughly the same time period, which no longer allows him to fit into his sneakers. He denies fever, chills, chest pain, palpitations, cough, pleurisy, calf pain, abdominal complaints, sick contacts, or travel. His psychosocial history is noteworthy for chronic alcohol use. His physical exam reveals bibasilar rales, JVD of 5 cm, an S3 gallop, a holosystolic murmur at the apex that radiates to the left axilla, and 2+ pitting edema to the level of the mid-calves bilaterally. A bedside echocardiogram was remarkable for biventricular enlargement. Question What additional physical exam finding would be expected in this patient? 1 Tachycardia 2 Fever 3 Asymmetric upper extremity blood pressures 4 Warm, moist skin 5 Acanthosis nigricans

1. Tachycardia

A 38-year-old woman with a past medical history of rheumatic fever and endocarditis presents with progressive dyspnea on exertion associated with palpitations and intermittent episodes of left-sided chest pain. Both symptoms resolve at rest. Her physical exam reveals resting tachycardia and a widened pulse pressure. The cardiac exam is notable for a decrescendo diastolic high-pitched murmur, loudest at the left sternal border and accentuated with the patient leaning forward in full expiration. Abrupt distention and quick collapse are observed upon palpation of the peripheral arterial pulses. Booming systolic and diastolic sounds are auscultated over the femoral arteries. Question What is the diagnostic test of choice to confirm this patient's most likely diagnosis? 1 Transthoracic echocardiography 2 Chest radiography 3 Cardiac computed tomography 4 Electrocardiography 5 Cardiac troponins

1. Transthoracic echocardiography

A 56-year-old man presents with a 1-week history of palpitations and shortness of breath. He has a longstanding history of poorly controlled hypertension. Physical examination reveals an elevated blood pressure of 190/98 mm Hg, elevated jugular venous pressure, mild hepatomegaly, bilateral pedal edema, and rales at the lung bases. Echocardiogram reveals concentric left ventricular hypertrophy without significant valvular abnormalities. Question What drug is beneficial in the treatment of the patient's condition by causing afterload and preload reduction? 1 Loop diuretic 2 Angiotensin-converting enzyme inhibitor 3 Positive inotropic agent 4 Sodium channel blocker 5 Arterial vasodilator

2. Angiotensin-converting enzyme inhibitor

A 61-year-old man presents with a recent history of increased fatigue with mildly increased exertional dyspnea. The patient denies any significant past medical history but states that he had some heart problems as a child; he was never clear as to what the problem was. On cardiac examination, you hear an early diastolic soft decrescendo murmur with a high-pitched quality, especially when the patient is sitting and leaning forward. No thrill is felt. Question What is the most likely diagnosis? 1 Tricuspid stenosis 2 Aortic regurgitation 3 Mitral stenosis 4 Mitral valve prolapse 5 Pulmonic stenosis

2. Aortic regurgitation

A 51-year-old man with a history of Marfan syndrome presents to the emergency department with tearing chest pain that radiates to the back and neck. On examination, the patient is hypertensive; a high-pitched decrescendo diastolic murmur at the left sternal border and diminished peripheral pulses are noted. Question Based on the patient's most likely diagnosis, what is the immediate diagnostic imaging modality of choice? 1 Posterior-anterior chest X-ray (PA CXR) 2 Computed tomography scan of the chest and abdomen (CT scan) 3 Magnetic resonance imaging of the chest (MRI) 4 12-lead electrocardiogram (ECG) 5 Transesophageal echocardiography (TEE)

2. Computed tomography scan of the chest and abdomen (CT scan)

A 73-year-old man presents with worsening shortness of breath with activity over the last few months. He is not able to complete as many physical activities during the day as he could manage 3 months ago. Physical examination reveals hypotension, tachycardia, extremities that are cool to the touch, expiratory wheezing, and rhonchi auscultated during the pulmonary exam; there is also a diminished first heart sound with an S3 gallop heard during the cardiac exam. Question You order an ECG, but before receiving the results, you are highly suspicious that the patient has what diagnosis? 1 Pulmonary embolism 2 Congestive heart failure 3 Myocardial infarction 4 First-degree heart block 5 Pulmonary hypertension

2. Congestive heart failure

A 52-year-old Caucasian man who frequently visits the emergency department presents due to "being short of breath." He is currently homeless, in and out of shelters. Past presenting problems that have brought him to the emergency department indicate that he has a chronic issue with alcohol overuse. Today, the patient is experiencing severe shortness of breath at rest. Physical examination findings reveal rales auscultated in bilateral lung fields, an S3 gallop, and elevated JVP. Question What is the most likely diagnosis? 1 Angina pectoris 2 Dilated cardiomyopathy 3 Essential hypertension 4 Hypertrophic cardiomyopathy 5 Restricted cardiomyopathy

2. Dilated cardiomyopathy

An 82-year-old man with a past medical history of hypertension, dyslipidemia, type 2 diabetes, and chronic kidney disease is being evaluated for progressive exercise-induced fatigue and shortness of breath over the last year. He also admits to more recent chest pain and lightheadedness, both of which occur with ambulation. He denies cough, fever, chills, lower extremity edema, or abdominal complaints. The physical exam revealed a narrow pulse pressure following blood pressure assessment. His cardiac exam noted a laterally displaced point of maximal impulse, as well as a mid-systolic ejection murmur that is low-pitched, rough, rasping in character, and loudest in the second right intercostal space. This murmur radiates to the bilateral carotid arteries. His peripheral vascular exam demonstrated a delayed peak of his radial pulsations. Question What diagnostic procedure would be best for this patient? 1 EKG 2 Echocardiography 3 Lateral chest X-ray 4 Cardiac catheterization 5 Immediate coronary angiography

2. Echocardiography

A 25-year-old man presents after experiencing pronounced shortness of breath while at the gym. You cannot obtain satisfying information from him during the interview, and you think that he may have an intellectual disability disorder. On examination, he appears short for his age and you notice webbed neck, dental malocclusion, antimongoloid slanting of the eyes, and hypogonadism. Auscultation reveals high-pitched systolic ejection murmur maximal in the second left interspace with radiation to the left shoulder and ejection click that decreases with inspiration. His second heart sound (P2) is delayed and soft. The impulse of right ventricle is increased, and you palpate a thrill at second left intercostal space. Question What will be your best next diagnostic step? 1 Intellectual disability assessment 2 Echocardiography/Doppler 3 Cardiac catheterization 4 Cardiac MRI 5 Karyotyping

2. Echocardiography/Doppler

A 79-year-old man with a past medical history of coronary artery disease, diabetes mellitus, hypertension, smoking, alcohol use, and hyperlipidemia presents with severe chest pain and dyspnea. He appears pale, apprehensive, and diaphoretic. He is in a confused state and agitated. His pulse is weak and tachycardic, with a systolic blood pressure of 60 mm Hg. He has a narrow pulse pressure, tachypnea, a weak apical impulse, and significant jugular venous distention. His lungs are free of crackles. Bedside electrocardiogram revealed anterolateral ST segment elevations and "tombstones" across the precordial leads. Question Following stabilization, what health maintenance advice should be provided to this patient? 1 Increase weight gradually. 2 Eliminate smoking and alcohol. 3 Increase dietary saturated fat and sodium. 4 Cease exercise programs. 5 Discontinue antihypertensives.

2. Eliminate smoking and alcohol.

A 39-year-old previously well Caucasian man presents to the emergency department with a 10-day history of fever >101°F and acute dyspnea with pleuritic chest pain. His past medical history is notable only for childhood asthma (no recurrences since age 12) and appendectomy. He has no known drug allergies. He denies taking prescribed medications on a regular basis. Vital signs show: Temperature 100.8°F, pulse 108, respirations 24, and blood pressure 98/60. O2 saturation is 90% on room air. Physical examination reveals mild crackles of the mid-lung fields bilaterally and a grade II/VI soft systolic murmur, loudest at the left lower sternal border. Oral exam shows overall poor dentition. Skin exam shows non healed puncture wound in left antecubital region surrounded by old granulomas and scarring. A spiral CT reveals evidence of multiple pulmonary emboli. He is admitted to the general medical floor of an acute care hospital. Additional diagnostic tests are ordered; preliminary results of blood cultures showed 4+ growth of gram-positive cocci. Infectious Diseases is consulted and he is started on an IV antibiotic regimen. Question What is the most likely causative risk factor for this patient's underlying diagnosis? 1 Dental infection 2 IV drug abuse 3 Kawasaki disease 4 Rheumatic heart disease 5 Valvular heart disease

2. IV drug abuse

A 55-year-old man presents with severe central chest pain. Pain started suddenly and it radiates to the back and neck. He is unable to lie flat. He feels sick but has not vomited. He has no major illnesses and knows of none that run in his family. He does not use alcohol, tobacco, or illicit substances. He is allergic to sulfa drugs. On physical exam, he appears in extreme pain and is lying on his side. Temperature is 98.6°F, heart rate is 110, blood pressure is 180/105 mm Hg, and respiratory rate is 20. Cardiac exam reveals normal S1 and S2 without rubs or gallop. The top of his internal jugular venous column is present at 2-3 cm above the sternal notch. Chest auscultation shows normal vesicular breathing. He has normal active bowel sounds tympanic to percussion. Extremity exam is normal, and the lower motor and sensory function are intact. ECG shows left ventricular hypertrophy. Chest X-ray shows widened mediastinum. Question What treatment should be given immediately to this patient? 1 Nitroglycerin spray 2 Intravenous labetalol 3 Streptokinase 4 Angioplasty 5 Surgery

2. Intravenous labetalol

A 68-year-old man with a history of obesity, hypertension, hyperlipidemia, and myocardial infarction presents with a 3-day history of shortness of breath at rest, making it difficult to walk short distances. He also notes orthopnea, nocturnal dyspnea, and generalized abdominal discomfort. He denies cough, fever, chills, diaphoresis, anxiety, chest pain, pleurisy, nausea, vomiting, diarrhea, rashes, lightheadedness, and syncope. He is acutely dyspneic, afebrile, tachypneic, and diaphoretic. There is a diminished first heart sound, S3 gallop, and laterally displaced PMI; the lungs have bibasilar rales. Abdominal exam reveals distension with hepatomegaly in the right upper quadrant. There is 2+ pitting edema of the lower extremities to the level of the mid-calf. A chest X-ray reveals pulmonary vascular congestion. Question What health maintenance strategy should this patient employ? 1 Increased caloric intake for myocardial function 2 Monitoring of hypertension and valvular dysfunction 3 Sodium consumption for cardiovascular benefit 4 Preservation of ejection fraction 5 Intensive exercise regimen

2. Monitoring of hypertension and valvular dysfunction

A 45-year-old man is admitted to the hospital with fever, weakness, weight loss, extremity pain, and a rash on his legs. He states that his symptoms began about 1 week ago. He denies recent illness or injury and states that he has been in good health for as long as he can remember. On physical exam, the patient was well-developed, well-nourished, and in mild physical distress. His lower extremities have ulceration near his medial malleoli and a diffuse lace-like purplish discoloration on his bilateral lower extremities. His blood pressure was elevated at 152/94 mm Hg, and a chest radiograph was negative. Laboratory analysis revealed an elevated sedimentation rate and C-reactive protein, elevated BUN, and creatinine. His red blood cell count was decreased, and his ANCA was negative. Hepatitis B tests were also negative. Biopsy of the leg ulceration reveals a fibrinoid necrosis of an arterial wall with a leukocytic infiltrate. Question What is the most likely cause of this patient's symptoms? 1 Systemic lupus erythematosus 2 Polyarteritis nodosa 3 Rheumatoid arthritis 4 Sjögren syndrome 5 Kawasaki disease

2. Polyarteritis nodosa

A 62-year-old man with a 15-year history of hypertension presents with severe tearing chest pain radiating through to the back. Blood pressure is 180/110 mm Hg, heart rate is 120 bpm, and respiratory rate is 34/min. Physical examination findings include neck negative for bruits/JVD, lungs clear to auscultation, regular heart rhythm, normal S1/S2 with an S4 present, and a grade III/IV diastolic rumbling murmur noted with the patient leaning forward. Radial pulses are 1+ on right and 3+ on left. EKG reveals a sinus tachycardia and evidence of left ventricular hypertrophy. A STAT chest X-ray is obtained. Question What finding is most consistent with the presumptive diagnosis? 1 Prominent pulmonary hila 2 Widening of the superior mediastinum 3 Kerley B lines 4 Right-sided pulmonary effusion 5 Prominent right cardiac shadowing

2. Widening of the superior mediastinum

A 58-year-old man presents with worsening shortness of breath (SOB); it has been especially problematic over the last 4 months. He states that the SOB has been noticeably severe with exertion, and this has drastically decreased his ability to do any physical activities. He is now experiencing SOB at rest. Physical examination reveals mild abdominal distension secondary to ascites and 4+ bilateral peripheral lower extremity edema. Past medical history includes that the patient underwent radiation and chemotherapy for lung cancer about 7 years ago. Question Given the most likely diagnosis, what is a primary goal when utilizing pharmacologic agents for this patient? 1 Increase systemic congestion. 2 Increase ventricular filling pressure. 3 Decrease pulmonary congestion. 4 Decrease the systolic pump function. 5 Decrease cardiac output.

3. Decrease pulmonary congestion

A 54-year-old man presents with chest pain. He has a past medical history of hypertension and diabetes mellitus. The pain is located in the middle of his chest and radiates to his jaw. The pain began about 20 minutes ago, and he rates the pain as a 10 on a 0-10 point scale, with 10 being the worst pain he has ever felt. He has had 3 similar episodes, but they have always resolved after 5 minutes or so of rest. He has smoked 1 pack of cigarettes a day for the past 36 years. He drinks 2 or 3 beers on Friday nights. Review of systems (ROS) is positive for diaphoresis, acute dyspnea, and impending doom. ROS negative for fever, chills, and malaise. Physical exam shows an obese, middle-aged man in moderate distress. BP is 126/80 mm Hg, pulse is 100, and respirations are 26. Heart and lung exams are normal, except for tachycardia and tachypnea. He has no pedal edema. Question What aspect of the patient's history is the most influential risk factor for an acute myocardial infarction? 1 Alcohol use 2 Male sex 3 Diabetes mellitus 4 Hypertension 5 Obesity

3. Diabetes mellitus

A 55-year-old man presents with a 5-day history of sharp chest pain. He has a history of rheumatic fever that occurred when he was 15. Further questioning reveals that the patient has had a high fever along with this chest pain. The pain itself seems to vary with positioning and movement, but it extends into the shoulder; he gains mild relief sitting up, but when he is laying down, the pain becomes intense. Physical examination reveals a pericardial friction rub. You order an ECG to be performed. Question Considering the most likely diagnosis, what findings would confirm your suspicion? 1 ST segment depression 2 Lengthening PR intervals 3 Diffuse ST-segment elevation 4 P wave missing QRS complex 5 Peaked T waves

3. Diffuse ST-segment elevation

A 43-year-old man presents for a general physical exam. He states that he has no significant past medical history. On physical examination, you note that the patient has a 3/6 diastolic murmur; it is heard best in the right upper chest. Blood pressure is 152/62 mm Hg. No other abnormalities are noted. Question What is the next step to confirm diagnosis? 1 EKG 2 Coronary heart catheterization 3 Echocardiogram with color Doppler 4 Cardiac CT scan 5 Chest X-ray

3. Echocardiogram with color Doppler

A 71-year-old woman, previously in excellent health and taking no medications, presents with new shortness of breath when she lies down at night. She denies any difficulty breathing during the day and states she is able to maintain her usual level of light activity. She denies any chest pain or palpitations. She noticed some mild ankle edema around the time the respiratory symptoms started. Office spirometry is normal, but her EKG reveals a widened QRS complex and her laboratory results reveal an elevated BNP (brain naturietic peptide). Question What is the most likely diagnosis? 1 Asthma 2 COPD 3 Heart failure 4 Myocardial infarction 5 Viral URI

3. Heart failure

A 68-year-old woman presents with shortness of breath, fatigue, dry cough, and swelling in her ankles. Symptoms started around the time of her son's wedding 6 weeks ago. She dismissed them as stress-related, but symptoms have worsened in intensity and frequency. Now she becomes very short of breath with any exertion, such as climbing stairs in her home. She feels like she may pass out and has to sit when she gets lightheaded. Her breathing is more difficult when lying down. She denies productive cough, fever, or chest pain. She has had no medical care for several years. Her past medical history is unremarkable, with no known medical conditions and no surgeries. She is menopausal and does not take any medications, nor does she have any allergies. She denies the use of tobacco, alcohol, and drugs. Vitals are shown in the table. Physical exam was performed by the provider on an earlier shift, and the patient's care was turned over to you. Weight 171 lb Height 65" BMI 28.5 Pulse 101 Blood pressure 158/98 mm Hg Temperature 97.9°F/36.6°C Complete blood count (CBC) Normal Thyroid-stimulating hormone (TSH) Normal Urinalysis Normal Complete metabolic panel (CMP) Normal Brain natriuretic peptide (BNP) Elevated Chest X-ray Perivascular edema, slightly enlarged cardiac shadow 12-lead electrocardiogram Mild left ventricular hypertrophy Cardiac echocardiogram Pending Cardiac enzymes Normal Question What physical exam findings would be expected given this patient's history and test results? 1 Absent breath sounds over single lung lobe 2 Dullness to percussion over left upper lung fields 3 Hepatojugular reflux 4 Pericardial friction rub 5 Positive Homan sign

3. Hepatojugular reflux

A 55-year-old woman presents with bumps around her eyes. She states they have been worsening over the past 3-4 months. They are not painful, but she is worried they may be something serious, especially since they seem to be increasing in size. The patient is currently taking a regular dose aspirin, which was suggested by her gynecologist, but she is not on any other daily medications. Family history is pertinent for her father dying at age 82 due to a heart attack and her mother still living at the age of 79 with a known medical history of hypertension and high cholesterol. On physical examination, slightly raised yellowish well-circumscribed plaques along the nasal portion of both eyelids are noted. Question What is the most likely diagnosis? 1 Congestive heart failure 2 Diabetes mellitus 3 Hypercholesterolemia 4 Hypertension 5 Lupus

3. Hypercholesterolemia

A 70-year-old woman with a history of hypertension, hyperlipidemia, and myocardial infarction presents with a 3-day history of shortness of breath at rest. She has found it difficult to walk short distances due to shortness of breath and is experiencing orthopnea and nocturnal dyspnea. She denies cough, fever, chills, nausea, abdominal pain, vomiting, diarrhea, or rashes. Upon physical examination, the patient is short of breath, requiring numerous pauses during conversation. General assessment reveals the patient is tachycardic and diaphoretic Cool extremities. Heart exam reveals a diminished first heart sound, S3 gallop, and laterally displaced PMI. Lung exam reveals dullness to percussion, bibasilar rales, and expiratory wheezing. 2+ pitting edema of the lower extremities to the level of the mid-calf. There is no JVD noted. Question What intervention will provide the greatest symptomatic relief to this patient? 1 Pericardiocentesis 2 Intravenous beta blocker 3 Intravenous diuretic 4 Oral calcium channel blocker 5 Subcutaneous Lovenox

3. Intravenous diuretic

A 20-year-old male student presents with a "weird" heart sound. He indicates that while in the anatomy lab, they were practicing with stethoscopes and listening to each other's heart sounds as a part of the course curriculum. One student said that his heart had a rumbling sound. On further history of the patient, he indicated that he had acute rheumatic fever twice as a teen. Question Considering the most likely diagnosis, how is the pathology best auscultated for assessment? 1 Patient standing, at the apex of the heart 2 Patient sitting, at the base of the heart 3 Patient lying on the left side, at the apex of the heart 4 Patient leaning forward, at the base of the heart 5 During a Valsalva maneuver

3. Patient lying on the left side, at the apex of the heart.

A 26-year-old African American man with no significant past medical history presents with a history of dyspnea on exertion that occurs after running. The dyspnea is associated with mild substernal chest pain. All symptoms are relieved with rest. He denies fever, chills, cough, wheezing, pleurisy, calf pain, abdominal problems, peripheral edema, cigarette, drug, or alcohol use, sick contacts, or travel. His physical exam reveals a harsh murmur best heard at the left lower sternal border and an S4 gallop. A bedside electrocardiogram was remarkable for left ventricular hypertrophy and septal Q waves in the inferolateral leads. An echocardiogram noted asymmetric LVH, anterior motion of the mitral valve during systole, a small and hypercontractile LV, and delayed relaxation and filling of the LV during diastole. The septum was twice the thickness of the posterior wall. Question What is expected to be true regarding the murmur in this patient? 1 It decreases while in an upright posture. 2 It becomes less intense with the Valsalva maneuver. 3 It decreases with squatting. 4 It increases with sustained handgrip. 5 It increases with lying down.

3. It decreases with squatting

A 68-year-old woman presents with a several-month history of feeling dizzy and lightheaded. She admits brief syncope once. These episodes often occur when she gets out of bed and after dinner. She admits some general weakness but feels well. She denies seizures, headaches, numbness, paresthesias, and gait or balance disturbances. She did some routine labs for a health fair last month and reports all values returned normal. PMH is non-contributory. She is menopausal, with no medications, surgeries, or allergies. She is retired, lives with her husband, and does not exercise regularly. On physical exam, her blood pressure is taken both lying supine and then again 3 minutes later standing upright. Readings are 110/78 mm Hg supine and 82/63 mm Hg standing. Mucous membranes are moist, and skin turgor is good. The patient's physical exam, including cardiovascular, pulmonary, and neurological, are normal. Electrocardiogram is normal. Question Assuming the workup does not reveal an underlying cause of this patient's condition, what non-medication intervention is most appropriate? 1 Daily magnesium supplementation 2 Implantation of spinal cord stimulator 3 Liberal salt and fluid intake 4 One serving of red wine daily 5 Regular aerobic exercise

3. Liberal salt and fluid intake

An 80-year-old woman has a history of a myocardial infarction about 5 weeks ago. The patient now presents to the ED with a history of congestive heart failure with symptoms of shortness of breath and peripheral edema, which appear to be worsening since her MI. An ECG is conducted, which reveals a prior inferior wall infarct. Cardiology is consulted where a 2-dimensional echo is conducted, displaying severe leaflet tethering and an enlarged left ventricle. Question What structure is most likely to be affected? 1 Chordae tendineae 2 Interventricular septum 3 Mitral valve 4 Pulmonic valve 5 Tricuspid valve

3. Mitral valve

A 30-year-old woman with no significant past medical history presents with a history of recurrent palpitations. These episodes occur primarily upon exertion. She recalls periodic bouts of anxiety, panic attacks, and lightheadedness. She denies fever, chills, changes in weight, chest pain, shortness of breath, rashes, diaphoresis, abdominal pain, nausea, and vomiting. She denies any history of cigarette smoking, drug, or alcohol use. Her physical exam revealed normal vital signs. The cardiac exam revealed a high-pitch late systolic click at the apex. The valsalva maneuver and a standing position result in prolongation of the murmur and a movement of the click to earlier in the cardiac cycle. The remainder of her examination is normal. Question What is the most likely diagnosis? 1 Patent ductus arteriosus 2 Mitral regurgitation 3 Mitral valve prolapse 4 Mitral stenosis 5 Hypertrophic cardiomyopathy

3. Mitral valve prolapse

An otherwise asymptomatic, thin 18-year-old woman presents with a history of scoliosis and the presence of a mid-systolic click that is auscultated during physical examination. The patient most likely has what valvular disease? 1 Mitral stenosis 2 Aortic stenosis 3 Mitral valve prolapse 4 Pulmonic stenosis 5 Aortic valve prolapse

3. Mitral valve prolapse

A 45-year-old woman presents with chest pain. The pain developed about an hour prior to arrival in ED and was precordial. No positional or pleuritic component was present. Nitroglycerin relieved the pain. Her EKG reveals ST elevations. She undergoes cardiac catheterization; all of her vessels are clean and no revascularization is performed. Question Given the most likely diagnosis, what is the best pharmaceutical option for this patient? 1 Diazoxide 2 Metoprolol 3 Nifedipine 4 Atenolol 5 Ibuprofen

3. Nifedipine

A 65-year-old African American man with a past medical history of hypertension, hyperlipidemia, and diabetes experiences substernal chest pain while shoveling snow. The patient says the pain started after 10 minutes of shoveling wet snow and eventually resolved after he sat down and rested. The patient described the pain as a "heaviness" that did not radiate to any other part of his body and as a 4 or 5/10 on a subjective pain scale, and he experienced this discomfort for approximately 1-2 minutes total. The patient's current vital signs are blood pressure 168/98 mm Hg, pulse 92, and respirations 16. Question What drug would be the best choice to rapidly reduce the patient's chest pain in a future similar situation? 1 Aspirin 81 mg PO 2 Metoprolol 50 mg PO 3 Nitroglycerin 0.4 mg SL 4 Simvastatin 10 mg PO 5 Lisinopril 5 mg PO

3. Nitroglycerin 0.4 mg SL

A 34-year-old woman presents with a 2-week history of severe fatigue, increased swelling in both feet, and slight pain in the right abdomen. She gives a history of shortness of breath on severe exertion. Exam reveals an afebrile patient with pedal edema and hepatomegaly. There are prominent A waves of the jugular venous pulsations (JVP). Auscultation reveals a tricuspid opening snap. A diastolic murmur is heard over the left sternal border, which increases on inspiration. A widely split S1 is also heard. Diagnostic testing reveals a normal CBC. Right atrial enlargement is seen on the chest X-ray, and the echocardiogram shows thickened tricuspid leaflets with limited mobility, increased velocity rough diastolic flow, and prolonged pressure half-time. Question What is the cause of hepatomegaly in this patient? 1 Metastasis of tumor from the right atrial sarcoma 2 Primary hepatic tumor 3 Obstructed venous flow to the right ventricle 4 Backflow pressure due to inferior vena cava obstruction 5 Acute infection of the liver by hepatitis B virus

3. Obstructed venous flow to the right ventricle

A 39-year-old man presents with a 1-week history of severe chest pain. He states that the pain seems to worsen when he lies down. He describes the pain as radiating to the back and worsening when he takes a deep breath. His vital signs are as follows: blood pressure 124/84 mm Hg, respiratory rate 18/min, temperature 101°F, and pulse 74 beats per minute. On auscultation of the chest, you cannot distinguish an S1 or S2 but hear a scratching or grating sound. Question What is the first step in the treatment of this patient? 1 Pericardiocentesis 2 Beta-blockers 3 Rest and NSAIDs 4 Corticosteroids 5 Calcium channel blockers

3. Rest and NSAIDs

A 54-year-old man presents with a 6-month history of increasing intolerance to exercise. He describes "breathlessness" with exertion, as well as fatigue and 2-pillow orthopnea. He denies tobacco use but does admit to 4 or 5 whiskey sours daily for the last 20 years. He is a businessman and often entertains clients, which "involves drinking alcohol." Chest X-ray reveals an enlarged cardiac silhouette. EKG reveals normal sinus rhythm. A surface echocardiogram reveals an ejection fraction of 35%, mild mitral regurgitation, and dilated left ventricle. Question What statement about this patient's illness is true and should be shared with the patient? 1 His cardiomyopathy is likely familial. 2 There is minimal risk with this disease, so he can be treated symptomatically. 3 Symptoms can significantly improve with alcohol cessation. 4 He is in need of an implantable defibrillator emergently. 5 His cardiomyopathy is the result of his heart murmur.

3. Symptoms can significantly improve with alcohol cessation

A 62-year-old man with a 15-year history of hypertension presents with severe tearing chest pain radiating through to the back. His blood pressure is 180/110 mm Hg, heart rate is 120 BPM, and respiratory rate is 34/min. Physical exam findings include lungs clear to auscultation, heart regular rhythm, normal S1/S2 with an S4 present, and a grade III/IV diastolic rumbling murmur noted with the patient leaning forward. Radial pulses are 1+ on right and 3+ on left. No evidence of JVD and carotids are negative for bruits. EKG reveals a sinus tachycardia and evidence of left ventricular hypertrophy. Question What is the most likely diagnosis? 1 Spontaneous pneumothorax 2 Pulmonary saddle embolus 3 Thoracic aortic dissection 4 Coarctation of the aorta 5 Acute viral pericarditis

3. Thoracic aortic dissection

A 72-year-old man with a history of poorly controlled HTN and previous myocardial infarction presents with a nocturnal cough, bilateral ankle swelling, and dyspnea on exertion. He denies any fever, chills, URI symptoms, chest pain, headache, N/V, diaphoresis, or syncope. He further denies smoking, alcohol, or drug use. Physical exam reveals bipedal edema and bibasilar crackles. A chest X-ray is remarkable for enlargement of the cardiac silhouette and interstitial infiltrates, while EKG analysis indicates deep S waves in lead V1 and tall R waves in lead V5. Question What is the most appropriate next step in the evaluation and management of this patient? 1 Biopsy of the endocardium 2 Chest CT scan with IV contrast 3 Transthoracic echocardiography 4 Bronchoalveolar lavage 5 Coronary angiography

3. Transthoracic echocardiography

A 56-year-old man is hospitalized with sudden onset of symptoms of chest pain, sweating, palpitation and shortness of breath. ECG showed ST elevation of 3 mm above isoelectric ECG line, and troponin I of 6 ng/mL. His BP is 130/75 mm Hg, and HR is 65 bpm. The next morning, Doppler and transesophageal echocardiography were performed following new onset of chest pain, shortness of breath and systemic hypotension, which established mitral regurgitation with papillary muscle rupture. Question What would be the most appropriate therapeutic intervention at this point? 1 Increase the dose of nitrates. 2 Initiate treatment with a beta blocker. 3 Perform surgery to insert a ventricular assist device. 4 Perform mitral valvuloplasty. 5 Postpone the intervention until acute phase is over.

4. Perform mitral valvuloplasty

A 70-year-old woman presents with shortness of breath at rest over the past 3 days. She has found it difficult to walk short distances due to shortness of breath. Additionally, she is experiencing confusion, orthopnea, nocturnal dyspnea, and lightheadedness. She denies cough, fever, chills, diaphoresis, anxiety, chest pain, pleurisy, nausea, abdominal pain, vomiting, diarrhea, rashes, and syncope. On physical examination, the patient is short of breath, requiring numerous pauses during conversation. She is afebrile, but she is tachycardic, diaphoretic, and her extremities are cool. There is a diminished first heart sound, S3 gallop, laterally displaced PMI, bibasilar rales, and dullness to percussion and expiratory wheezing noted. An elevated JVD and 2+ pitting edema of the lower extremities is evident. Question What statement regarding this patient's condition is correct? 1 The most common etiology of this condition is infiltrative diseases. 2 Increased caloric and sodium intake improves patient outcomes. 3 Confusion is the most common presenting complaint in older patients. 4 Activation of the renin-angiotensin-aldosterone system occurs. 5 The electrocardiogram is the most useful diagnostic test.

4. Activation of the renin-angiotensin-aldosterone system occurs.

A 63-year-old woman presents with a 1-hour history of left shoulder pain and nausea. She has a past medical history of coronary artery disease and had a stent placed 5 years ago. Troponin is elevated. An ECG shows large R waves and ST segment depression in leads V1, V2, and V3. Question These ECG findings are most consistent with what condition? 1 Acute ischemia without myocardial infarction 2 Acute lateral myocardial infarction 3 Acute inferior myocardial infarction 4 Acute posterior myocardial infarction 5 Acute anterior myocardial infarction

4. Acute posterior myocardial infarction

A 42-year-old man presents with a 3-day history of intermittent sharp stabbing chest pain that has become progressively worse and more continuous over the past 24 hours. The pain is aggravated by deep breathing and lying flat; it is relieved by sitting and leaning forward. He has no previous cardiac history. Upon further questioning, he mentions that he recently "had the flu." Physical examination reveals an anxious patient in moderate distress. Vital signs are as follows: BP 138/90 mm Hg Left Arm, HR 104 bpm and regular, RR 18/min and shallow, T 99.8°F. Normal breath sounds and a pericardial friction rub are noted upon auscultation of the chest. A stat ECG reveals diffuse ST elevation. Question What therapy should be initiated for this patient? 1 Thrombolytic agents 2 IV antibiotics 3 Immediate pericardiocentesis 4 Anti-inflammatory medications 5 Sublingual nitroglycerin

4. Anti-inflammatory medications

A 70-year-old Latinx man with a past medical history of hyperlipidemia presents with recurrent chest pain that has been occurring over the past month. This pain is provoked with activity, especially of his upper torso and left arm. The pain is described as sharp and generally located in the left pectoral area. He denies any weight changes, fever, or chills. He also denies cough, wheezing, pleurisy, calf pain, wheezing, vomiting, diarrhea, changes in bowel habits, cigarette, drug or alcohol use, sick contacts, and travel. Physical exam reveals an elderly Latinx man in no acute distress. Vital signs, skin, cardiopulmonary, abdominal, and peripheral vascular exams are found to be within normal limits. Question What additional historical or physical exam finding would suggest a non-ischemic etiology of this patient's chest pain? 1 Pain associated with anxiety, nausea, shortness of breath, and syncope 2 Pain relief upon administration of nitrate drugs 3 Associated history of diabetes mellitus and hypertension 4 Chest pain that is reproduced with palpation 5 Quality of pain described as a pressing, tightness, or heaviness

4. Chest pain that is reproduced with palpation

A 78-year-old woman is an inpatient status post-colectomy for colon cancer. On postoperative day 3, her oral temperature is noted to be elevated to 100.6°F. Chest X-ray and urinalysis are both negative for signs of infection. An infectious disease consult is placed in order to better define the patient's new fever. You suspect superficial thrombophlebitis. Question What physical examination findings would support your suspicion? 1 Abdominal tenderness in the right upper quadrant 2 Positive Homans sign in left calf 3 Abdominal tenderness in the left lower quadrant 4 Erythema and tenderness along the vein with IV insertion 5 Erythema and tenderness along her incision site

4. Erythema and tenderness along the vein with IV insertion

A 37-year-old overweight woman presents for a follow-up after an ER visit 4 days ago; her ER visit was the result of a fainting episode. Except for a blood pressure of 140/100, vital signs are all within normal limits. The patient denies a prior history of hypertension. Although the patient had an elevated BP, the ER physician felt that her fainting episode was more likely due to stress and poor eating habits; she had been attempting to lose weight before her wedding, which is occurring in a month. She was released from the ER with instructions to follow up with her PCP for evaluation and treatment of high blood pressure. Since this is her first visit to your clinic, a complete blood workup is done. Test Result Normal Range WBC 4.0 x 103/mcL 4.8-10.8 x 103/mcL RBC 5.3 x 103/mcL 4.7-6.1 x 103/mcL Hgb 13.5 g/dL 12.0-15.5g/dL Hct 40% 35-45% ALT 12 units/L 7-56 units/L AST 14 units/L 0-35 units/L Alkaline phosphatase 86 units/L 41-133 units/L BUN 10 mg/dL 8-20 mg/dL Creatinine 1.0 mg/dL 0.6-1.2 mg/dL Triglycerides 250 mg/dL <150 mg/dL Cholesterol 189 mg/dL <200 mg/dL TSH 2 mcU/mL 0.4-6 mcU/mL Question What medical condition does this patient have in addition to hypertension? 1 Anemia 2 Hyperthyroidism 3 Hypercholesterolemia 4 Hypertriglyceridemia 5 Leukocytosis

4. Hypertriglyceridemia

A 78-year-old man with known left-sided congestive heart failure presents due to cough, worsening dyspnea with exertion, and orthopnea. What is the most direct cause of his symptoms? 1 Tricuspid insufficiency 2 Left ventricular hypertrophy 3 Decreased peripheral vascular resistance 4 Increased pulmonary venous pressure 5 Mucus plugging

4. Increased pulmonary venous pressure

A 75-year-old man has a past medical history significant for hypertension, hyperlipidemia, stable angina, insulin-dependent type 2 diabetes, and nicotine dependence. Over the past 2 weeks, he has noticed progressively worsening shortness of breath following moderate exertion. While performing his regular morning routine today, he experienced acute onset substernal chest pain that lasted 30 minutes but gradually improved with rest. He presents to the emergency department soon after, accompanied by his wife. Vital signs obtained on triage are blood pressure 150/98 mm Hg, heart rate 87 BPM, SpO2 98%. Following initial evaluation, you obtain an ECG (Refer to the image). Laboratory studies obtained on arrival are remarkable for undetectable troponin I, procalcitonin, and comprehensive metabolic panel within normal limits. Question What treatment would be most appropriate for this patient? 1 Activate code STEMI and transfer to catheterization laboratory. 2 Begin treatment with NSAIDs and colchicine. 3 Discharge patient from emergency department with close follow-up. 4 Initiate dual antiplatelet and anticoagulant therapy. 5 Urgently administer intravenous anti-hypertensive medication.

4. Initiate dual antiplatelet and anticoagulant therapy.

A long-standing patient of yours is experiencing acutely worsening symptoms and signs of dilated cardiomyopathy. Symptoms include fatigue, dyspnea with mild exertion, paroxysmal nocturnal dyspnea, severe lower extremity edema, and clubbing. Physical exam is significant for an S3 gallop and jugular venous distention. Daily medications include a β-blocker, adult dose aspirin, and an ACE inhibitor. Question What clinical intervention would be the most logical next step in helping resolve their current symptoms? 1 Initiation of a calcium channel blocker 2 Initiation of an aldosterone antagonist 3 Initiation of a statin 4 Initiation of a diuretic 5 Initiation of an angiotensin II antagonist

4. Initiation of a diuretic

A 45-year-old woman presents with vision loss. The patient states that she was watching TV the other day and experienced vision loss in her right eye for a few minutes. She describes the loss as a curtain being brought down over the right eye; it stayed there for a few minutes and then lifted back up. Question In what artery is the etiology of this symptom located? 1 Anterior cerebral artery 2 Middle cerebral artery 3 Posterior cerebral artery 4 Internal carotid artery 5 External carotid artery

4. Internal carotid artery

A 37-year-old Caucasian man presents with shortness of breath. History reveals that the patient has been extremely fatigued the last few weeks, experiencing excessive night sweats with a worsening cough, chest pain, and general aches and pains. He is not taking any medications and is allergic only to penicillin. He has a history of on-and-off intravenous drug use and admits to last using around 1 month ago. Along with an urgent inpatient admission, you plan to initiate orders to have the patient undergo an echocardiogram and obtain blood cultures, among other actions. Question Based on the most likely diagnosis, what pharmaceutical intervention is most appropriate? 1 Intravenous corticosteroids 2 Intravenous cefazolin 3 Intravenous nafcillin 4 Intravenous vancomycin 5 Intravenous clindamycin

4. Intravenous vancomycin

A 27-year-old man with Marfan syndrome presents due to exercise intolerance and heart palpitations. On exam, you note a mid-systolic click and late systolic murmur heard at the apex of the heart. The click and murmur are noted later in systole with squatting and earlier in systole with sudden standing. Question What is the most likely diagnosis? 1 Aortic stenosis 2 Aortic regurgitation 3 Mitral stenosis 4 Mitral valve prolapse 5 Aortic dissection

4. Mitral valve prolapse

A 30-year-old woman presents for routine analysis of cholesterol levels. The results show plasma cholesterol levels of 300 mg/100 mL. You prescribe the drug simvastatin (Zocor). She is reluctant to take drugs to treat her hypercholesterolemia. After further discussion, she agrees to take a vitamin to treat the elevated cholesterol. She also has questions concerning familial hypercholesterolemia. Question What vitamin is effective in reducing circulating cholesterol levels when given in pharmacological doses? 1 Riboflavin 2 Thiamin 3 Vitamin K 4 Nicotinic acid 5 Pantothenic acid

4. Nicotinic acid

A 62-year-old woman presents with extreme fatigue and shortness of breath. The symptoms began about 24 hours ago and have progressively worsened within the last 4 hours. Vital signs on arrival are as follows: HR 90 beats per minute; BP 165/72 mm Hg; RR 16/min; SpO2 98% on 4 L/min supplemental oxygen by nasal cannula. 12-lead ECG demonstrates ST-segment elevation of 2 mm in leads V4-V6. Question In addition to an aspirin tablet, what medication would be most appropriate in the emergency management of this patient? 1 Dobutamine 2 Dopamine 3 Morphine 4 Nitroglycerin 5 Vasopressin

4. Nitroglycerin

A 72-year-old man presents for worsening shortness of breath, orthopnea, and chest pain for the last few weeks. He admits some chronic heart problems, fatigue, dyspnea, and non-productive cough, but he feels like symptoms have worsened recently. He denies fever, chills, and productive cough. On physical exam, he has mildly increased respiratory effort but does not appear in distress. He is barrel-chested. Breath sounds are diminished bilaterally, with dullness to percussion over right and left lower lungs. No pleural friction rub noted. S3 gallop, mild tachycardia (110 bpm), clubbing of the fingers, dependent edema in the lower extremities, and jugular venous distention are noted. His cardiac enzymes and electrocardiogram demonstrate no acute cardiac pathology. Pleural fluid and cardiomegaly are found on chest X-ray. Question What is the most plausible explanation for this patient's dependent edema? 1 Absence of venous valves leading to venous reflux 2 Hepatic fibrosis and decreased protein production with increased portal resistance 3 Large renal loss of protein leading to hypoalbuminemia and edema 4 Renal sodium and fluid retention leading to increased capillary pressure 5 Suppression of antidiuretic hormone release

4. Renal sodium and fluid retention leading to increased capillary pressure.

A 76-year-old man presents with substernal chest pain; it is associated with progressive exertional dyspnea, easy fatigability, and dizziness. These symptoms are exacerbated by walking short distances, and they are relieved with rest. He denies fever, chills, cough, wheezing, pleurisy, calf pain, abdominal problems, peripheral edema, cigarette, drug use, alcohol use, sick contacts, or travel. His physical exam reveals a normal blood pressure and a rough, harsh, low-pitched crescendo-decrescendo systolic murmur beginning after the first heart sound; it is best heard at the second intercostal space in the right upper sternal border. Its intensity is increased toward midsystole; the murmur radiates to both carotid arteries and is accentuated upon squatting, and it is reduced during Valsalva strain. His lungs are without adventitious sounds. Question What is correct regarding the management of this patient? 1 β-adrenergic blocker therapy is contraindicated in this patient. 2 An increase in aerobic activity to improve conditioning is recommended. 3 Maximum reduction of preload and afterload with ACE inhibitors is necessary. 4 Surgical intervention provides the only definitive treatment. 5 Bacterial endocarditis prophylaxis is required in patients with this diagnosis.

4. Surgical intervention provides the only definitive treatment

A 54-year-old man presents with chest pain. He has a past medical history of hypertension and diabetes mellitus. The pain is located in the middle of his chest and radiates to his jaw. The pain began about 20 minutes ago, and he rates the pain as a 10 on a 0-10 point scale, with 10 being the worst pain he has ever felt. He has had 3 similar episodes, but they have always resolved after 5 minutes or so of rest. He has smoked 1 pack of cigarettes a day for the past 36 years. He drinks 2 or 3 beers on Friday nights. Review of systems (ROS) is positive for diaphoresis, acute dyspnea, and sense of impending doom. ROS is negative for fever, chills, and malaise. Physical exam shows an obese, middle-aged man in moderate distress. BP is 148/80, pulse is 100, and respirations are 26. Except for tachycardia and tachypnea, heart and lung exams are normal. He has no pedal edema. Electrocardiogram (ECG) shows ST elevation in leads II, III, and AVF; this is a new finding when compared to an ECG from 3 months ago. Question What is the most likely diagnosis? 1 Pericarditis 2 Stable angina 3 Pulmonary embolism 4 Costochondritis 5 Acute myocardial infarction

5. Acute myocardial infarction

A 62-year-old man with a 15-year history of hypertension presents with severe tearing chest pain radiating through to the back. Blood pressure is 180/110 mm Hg, heart rate 120 bpm, and respiratory rate 34/min. Physical examination findings include lungs clear to auscultation, heart regular rhythm, normal S1/S2 with an S4 present, and grade III/IV diastolic rumbling murmur noted with the patient leaning forward. Radial pulses are 1+ on right and 3+ on left. Neck is negative for JVD and no carotid bruits present. EKG reveals a sinus tachycardia and evidence of left ventricular hypertrophy. A STAT chest X-ray shows a widening of the mediastinum. Question What medication class delivered intravenously is now needed to stabilize the patient? 1 Sympathomimetic drug 2 Positive inotropic drug 3 Alpha blocker 4 ACE inhibitor 5 Beta blocker

5. Beta blocker

A 74-year-old Caucasian man presents with progressive exertional shortness of breath, lower extremity edema, and lightheadedness over the previous 7 months. He has a past medical history of HIV, hepatic cirrhosis secondary to chronic alcohol abuse and hemochromatosis, obesity, and thiamine deficiency. His symptoms are improved with rest. The physical examination reveals rales, an elevated JVP, cardiomegaly, S3 gallop rhythm, a high-pitched, blowing holosystolic murmur at the apex, peripheral edema, and abdominal distension suggestive of ascites. A bedside EKG noted sinus tachycardia with nonspecific ST-T wave changes and Q waves. An echocardiogram of the patient is shown in the image. Question What is correct regarding the clinical intervention for this patient? 1 Calcium channel blockers and digoxin are considered first-line agents 2 This patient meets cardiac transplantation indications 3 The target systolic blood pressure in this patient is 140 mmHg 4 Oral anticoagulation is essential at this time 5 Beta-blockers have demonstrated reductions in cardiovascular mortality

5. Beta-blockers have demonstrated reductions in cardiovascular mortality

A 40-year-old obese Caucasian man says that his father recently passed away after having a heart attack. He is worried and wants to know whether he is at risk for cardiovascular disease. He is hypertensive; however, he does not take any medications. His BP in the office today is 140/96 mm Hg, and his BMI is 31. You explain to him that the risk factors for cardiovascular disease may be non-modifiable or modifiable. Question What is the modifiable risk factor for CAD in this man? 1 Age 2 Ethnicity 3 Sex 4 Family history 5 Hypertension

5. Hypertension

A 64-year-old man with a history of a remote myocardial infarction and congestive heart failure presents for his 3-month follow-up. A recent echocardiogram reveals severe left ventricular dysfunction. Question What intervention has been shown to reduce the risk of sudden cardiac death in similar patients? 1 Long-term calcium channel blocker therapy 2 Chronic daily lidocaine therapy 3 Daily administration of digitalis 4 Magnesium oxide therapy 5 Implantation of a cardioverter-defibrillator device

5. Implantation of a cardioverter-defibrillator device

A 42-year-old man with a past medical history of hypertension presents with a 6-week history of intermittent fever. He has an associated cough, dyspnea, anorexia, arthralgias, abdominal pain, diarrhea, a widespread rash throughout his body, and back pain. He has come to see you because he has experienced painless hematuria since this morning. The patient admits to a dental extraction approximately 6 weeks ago. He denies chills, a history of travel, sick or confined contacts, exposure to animals, bites, stings, cigarette smoking, otalgia, sore throat, swollen glands, drug use, dysuria, preceding GI or GU infections, previous surgeries, or sexual contact in the past year. Physical exam is remarkable for low-grade fever of 101°F, a generalized petechial rash and petechiae of the mucous membranes, dark red linear lesions of the nailbeds, tender subcutaneous nodules of the digital pads, and nontender maculae on the palms and soles. His heart is notable for a new harsh, medium pitched pansystolic murmur at the apex with radiation to axilla, and splenomegaly. Question What is the most likely diagnosis? 1 Reactive arthritis 2 Syphilis 3 Systemic lupus erythematosus 4 Lyme disease 5 Infective endocarditis

5. Infective endocarditis

A 55-year-old man got drunk at a pub after a serious argument with his wife about his diet and habits (he is overweight and a heavy smoker). On the way back home, he felt sudden chest pain radiating to the left shoulder, shortness of breath, sweating, and anxiety. His ECG shows pathological Q wave and ST elevation. Laboratory results show that troponin levels are elevated. Question What caused the rise in troponin levels in this patient? 1 Fatty changes in myocardial cells 2 Increased synthesis of adhesion molecules 3 Coexisting chronic viral infection 4 Long-term destruction of alcohol 5 Injury to myocardial cell membrane

5. Injury to myocardial cell membrane.

A 19-year-old man is brought into the ED following a motor vehicle crash in which he, the driver, sustained blunt trauma to the anterior trunk from striking the steering wheel and dashboard. The patient is alert, short of breath, hypotensive, and complains of acute chest pain. On auscultation, muffled heart sounds are heard. Question What is your initial diagnosis? 1 Pericarditis 2 Acute myocardial infarction 3 Pulmonary edema 4 Tension pneumothorax 5 Pericardial tamponade

5. Pericardial tamponade

A 73-year-old man presents with worsening shortness of breath on activity over the last few months. Another issue he mentions is not being able to complete as many physical activities during the day as he could perform 3 months ago. He states he has to use at least three pillows to allow him to sleep at night; otherwise, he cannot breathe. Physical examination confirms the suspected diagnosis. Question What sign would be consistent with this patient's most likely diagnosis? 1 Diminished lower extremity pulses 2 Diminished second heart sound 3 Hypertension 4 Non-healing ulcer 5 S3 gallop

5. S3 gallop

A 55-year-old man presents for a follow-up on recent cholesterol tests. He has a past medical history of 2 coronary stents placed 3 days ago after presenting to the emergency department with chest pain and a 40 pack-year history of smoking (he quit smoking 2 years ago). He was not on any medications when he presented to the ER. Cholesterol tests reveal: total cholesterol 200 mg/dL; LDL 100 mg/dL; HDL 40 mg/DL; triglycerides 395 mg/dL. Question What is the most appropriate treatment to reduce future cardiovascular risk in this patient? 1 Fibrates 2 Fish oil 3 Nicotinic acid 4 Plasmapheresis 5 Statin

5. Statin

A 68-year-old man with a past medical history of hypertension, hyperlipidemia, cluster headaches, polymyalgia rheumatica, and type 2 diabetes mellitus presents due to a 2-day history of constant left-sided throbbing headache of moderate severity. He admits to associated symptoms, such as pain across his mandible when he eats, fever, fatigue, and muscle aches. Most alarming to the patient was a single episode of complete left eye blindness that lasted for 30 minutes but has since resolved. He denies extremity numbness, tingling, muscle weakness, incontinence, and changes in mental status. He further denies rhinorrhea, ocular discharge, nausea, and vomiting. The physical exam was remarkable only for a tender left scalp with a noticeable pulsation underlying the tender area. Question What is the most likely diagnosis? 1 Bell's palsy 2 Herpes zoster ophthalmicus 3 Cerebrovascular accident 4 Cluster headache 5 Temporal arteritis

5. Temporal arteritis

An 86-year-old man has been experiencing fatigue, leg swelling and loss of appetite. His exam shows dependent edema, hepatomegaly, and JVD. Diuretics have been given to the patient and have worsened his renal function. Question What structure is most likely being affected in this patient's diagnosis? 1 Chordae tendineae 2 Interventricular septum 3 Mitral valve 4 Pulmonic valve 5 Tricuspid valve

5. Tricuspid valve


Conjuntos de estudio relacionados

Climate change and greenhouse gases

View Set

ch 10 Assessing and Responding to Fraud Risks

View Set

The scanning electron microscope

View Set

Ch 9: Negligence & Strict Liability

View Set

UEXCEL Human Resource Management

View Set